You are on page 1of 216

CHAPTER – 13

AREA RELATED TO CIRCLES


Exercise – 13.1

Page No 13.12:
Question 1: Find the circumference and area of a circle of radius 4.2 cm.
ANSWER:

T IO S I
We know that the circumference C and area A of a circle of radius r are
NO A D
given by 𝐶 = 2𝜋𝑟 and 𝐴 = 𝜋𝑟² respectively.

CO N
A
Here, 𝑟 = 4.2 cm

PY
DO U A
So substituting the value of r in above formulas,
T
ED PR

Circumference of the circle


𝐶 = 2 × 𝜋 × 4.2 cm
C

22
=2× × 4.2 cm
©

= 26.4 cm
Area of the circle
𝐴 = 𝜋 × 4.2 × 4.2 cm²
22
= × 4.2 × 4.2 cm²
7

= 55.4 cm²

Page No 13.12:
Question 2: Find the circumference of a circle whose area is 301.84 cm2.
ANSWER:
Let r cm be the radius of the circle. Then
Area of a circle is
𝐴 = 𝜋𝑟 2 cm2
301.84 cm2 = 𝜋 × 𝑟 2
22
301.84 cm2 = × 𝑟2
7

𝑟 2 = 96.04cm2

T IO S
𝑟 = 9.8 cm

I
NO A D
We know that the Circumference of circle of radius r is

CO N
A
𝐶 = 2𝜋𝑟 cm

PY
So substituting the value of r in above formula
DO U A
T
22
𝐶 =2× × 9.8 cm
ED PR

= 61.6 cm
C
©

Page No 13.12:
Question 3: Find the area of a circle whose circumference is 44 cm.
ANSWER:
Let r be the radius of the circle. Then
Circumference of the circle
𝐶 =2×𝜋×𝑟
22
44 cm = 2 × ×r
7
44
44 cm = ×r
7

𝑟 = 7 cm
We know that the area of a circle of radius r is
𝐴 = 𝜋𝑟 2
Substituting the value of r in above formula
𝐴 = 𝜋 × 7 × 7 cm²
22
= × 49 cm²
7

= 154 cm²

T IO S I
NO A D Page No 13.12:

CO N
A
Question 4: The circumference of a circle exceeds the diameter by 16.8
cm. Find the circumference of the circle.

PY
DO U A

ANSWER:
T
ED PR

Let the radius of a circle be r cm, then diameter of circle is 2r cm and


Circumference is 𝐶 = 2𝜋𝑟 cm.
C

It is given that the circumference exceeds the diameter of circle by 16.8


©

cm.
So, circumference = 16.8 + diameter
2𝜋𝑟 = 16.8 + 2𝑟 cm
22
2× × 𝑟 = 16.8 + 2𝑟 cm
7

44𝑟 = 117.6 + 14𝑟 cm


30𝑟 = 117.6 cm
𝑟 = 3.92 cm
Now the circumference is
𝐶 = 2𝜋𝑟 cm
22
=2× × 3.92 cm
7

= 24.64 cm

Page No 13.12:
Question 5: A horse is tied to a pole with 28 m long string. Find the area
where the horse can graze. (Take π = 22/7).

T IO S
ANSWER:

I
We know that the horse is tied to a pole with 28 m long string. So the
NO A D
horse can graze the area of a circle of radius 28 m.

CO N
A
Area of circle is

PY
DO U A
𝐴 = 𝜋𝑟 2
T
22
ED PR

= × 28 × 28 m²
7

= 2464 m²
C
©

Hence the horse can graze 2464 m² area.

Page No 13.12:
Question 6: A steel wire when bent in the form of a square encloses an
area of 121 cm2. If the same wire is bent in the form of a circle, find the
area of the circle.
ANSWER:
Let a cm be the side of square. Then area of square is
𝑎2 = 121 cm2
𝑎 = √121 cm2
𝑎 = 11 cm
We have,
Length of wire = perimeter of square
= 4a cm
= 4 × 11 cm
= 44 cm

T IO S
Let the radius of circle be r cm. Then,

I
Circumference of circle = length of wire
NO A D
CO N
2𝜋𝑟 = 44 cm
A
22
2× × 𝑟 = 44 cm

PY
DO U A
7
T
𝑟 = 7 cm
ED PR

Now, we will calculate area of circle.


Area of circle = 𝜋𝑟 2 cm²
C
©

22
= × 7 × 7 cm²
7

= 154 cm²

Page No 13.12:
Question 7: The circumference of two circles are in the ratio 2: 3. Find
the ratio of their areas.
ANSWER:
Let the radius of two circles be 𝑟1 cm and 𝑟2 cm respectively. Then their
circumferences are 𝐶1 = 2𝜋𝑟1 cm and 𝐶2 = 2𝜋𝑟2 cm respectively and
their areas are 𝐴1 = 𝜋𝑟12 cm² and 𝐴2 = 𝜋𝑟22 cm² respectively.
It is given that,
𝐶1 2
=
𝐶2 3
2𝜋𝑟1 2
=
2𝜋𝑟2 3
𝑟1 2
=
𝑟2 3

Now we will calculate the ratio of their areas,

T IO S
𝐴1 𝜋𝑟12
=
𝐴2 𝜋𝑟22

I
NO A D
𝑟12

CO N
=
𝑟22
A
𝑟1 2

PY
DO U A
=( )
𝑟2
T
𝑟
ED PR

Substituting the value of 1,


𝑟2

𝐴1 2 2
C

=( )
𝐴2 3
©

4
=
9

Hence the ratio of their Areas is 4: 9.

Page No 13.12:
Question 8: The sum of the radii of two circles is 140 cm and the
difference of their circumferences is 88 cm. Find the diameters of the
circles.
ANSWER:
Let the radius of two circles be 𝑟1 cm and 𝑟2 cm respectively. Then their
circumferences are 𝐶1 = 2𝜋𝑟1 cm and 𝐶2 = 2𝜋𝑟2 cm respectively and
their areas are 𝐴1 = 𝜋𝑟12 cm² and 𝐴2 = 𝜋𝑟22 cm² respectively.
It is given that the sum of the radii of two circles is 140 cm and difference
of their circumferences is 88 cm. So,
𝑟1 + 𝑟2 = 140 cm ……(A)
𝐶1 − 𝐶2 = 88 cm

T IO S
2𝜋𝑟1 − 2𝜋𝑟2 = 88 cm

I
2𝜋(𝑟1 − 𝑟2 ) = 88 cm
NO A D
CO N
88
A
𝑟1 − 𝑟2 = cm
2𝜋

PY
88
DO U A
𝑟1 − 𝑟2 = 22 cm
2× 7
T
ED PR

88×7
𝑟1 − 𝑟2 = cm
44

……(B)
C

𝑟1 − 𝑟2 = 14 cm
©

Now, solving (A) and (B)


𝑟1 = 77 cm
𝑟2 = 63 cm
Thus diameters of circles are,
2𝑟1 = 154 cm
2𝑟2 = 126 cm

Page No 13.12:
Question 9: Find the radius of a circle whose circumference is equal to
the sum of the circumference of two circles of radii 15 cm and 18 cm.
ANSWER:
Let the radius be r cm.
The radius of circle A, rA= 15 cm
The radius of circle A, rB = 18
Circumference of circle = circumference of circle A + circumference of
circle B
⇒ 2πr = 2πrA + 2πrB
⇒ r = rA + rB

T IO S
⇒ r = 15 + 18 = 33 cm

I
NO A D
CO N
A
Page No 13.12:

PY
Question 10: The radii of two circles are 8 cm and 6 cm respectively.
DO U A
Find the radius of the circle having its area equal to the sum of the areas
T
ED PR

of the two circles.


ANSWER:
C

Let the radius of circles be 𝑟 cm, 𝑟1 cm and 𝑟2 cm respectively. Then their


©

areas are 𝐴 = 𝜋𝑟 2 cm² , 𝐴1 = 𝜋𝑟12 cm²and 𝐴2 = 𝜋𝑟22 cm² respectively.


It is given that,
Area A of circle = Area 𝐴1 of circle + Area 𝐴2 of circle
𝜋𝑟 2 = 𝜋𝑟12 + 𝜋𝑟22
𝜋𝑟 2 = 𝜋(𝑟12 + 𝑟22 )
𝑟 2 = 𝑟12 + 𝑟22
𝑟 2 = 𝑟12 + 𝑟22
We have, 𝑟1 = 6 cm and 𝑟2 = 8 cm
Substituting the values of 𝑟1 , 𝑟2
𝑟2 = 6 × 6 + 8 × 8
𝑟 2 = 36 + 64
𝑟 2 = 100
𝑟 = √100
𝑟 = 10 cm
Hence, the radius of circle is 10 cm.

T IO S I
Page No 13.12:
NO A D
CO N
Question 11: The radii of two circles are 19 cm and 9 cm respectively.
A
Find the radius and area of the circles which has it circumference equal to

PY
DO U A
the sum of the circumferences of the two circles.
T
ANSWER:
ED PR

Let the radius of circles be 𝑟 cm, 𝑟1 cm and 𝑟2 cm, respectively. Then their
circumferences are 𝐶 = 2𝜋𝑟 cm, 𝐶1 = 2𝜋𝑟1 cm and 𝐶2 = 2𝜋𝑟2 cm
C
©

respectively.
It is given that,
Circumference C of circle
= Circumference 𝐶1 of circle + Circumference 𝐶2 of circle
2𝜋𝑟 = 2𝜋𝑟1 + 2𝜋𝑟2
2𝜋𝑟 = 2𝜋(𝑟1 + 𝑟2 )
𝑟 = 𝑟1 + 𝑟2
We have, 𝑟1 = 19 cm and 𝑟2 = 9 cm
Substituting the values of 𝑟1 , 𝑟2
𝑟 = 19 + 9
𝑟 = 28 cm
Hence the radius of the circle is 28 cm.
We know that the area A of circle is
𝐴 = 𝜋𝑟 2
Substituting the value of r
22
𝐴= × 28 × 28
7

T IO S
= 2464 cm²

I
Hence the area of the circle is 2464 cm².
NO A D
CO N
A

PY
Page No 13.12:
DO U A
T
Question 12: The area of circular playground is 22176 m2. Find the cost
ED PR

of fencing this ground at the rate of ₹50 per metre.


ANSWER:
C
©

Area of circular playground = 22176 m2


⇒ 𝜋𝑟 2 = 22176
22 2
⇒ 𝑟 = 22176
7
22176×7
⇒ 𝑟2 =
22

⇒ 𝑟 2 = 7056
⇒ 𝑟 = 84 cm
22
circumference = 2𝜋𝑟 = 2 × × 84 = 528 cm2
7
Cost of fencing the ground = 528 × 50 = Rs 26400
Page No 13.12:
Question 13: The side of a square is 10 cm. Find the area of circumscribed
and inscribed circles.
ANSWER:
It is given that the side of square is 10 cm.
So, the diameter of circle inscribed the square is 10 cm.
We know that the area A of circle inscribed the square is

T IO S
𝐴 = 𝜋𝑟 2

I
NO A D
Substituting the value of radius of inscribed circle 𝑟 = 5 cm,

CO N
A
𝐴 = 3.14 × 5 × 5

PY
DO U A
= 78.5 cm²
T
ED PR

Hence the area of circle inscribed the square is 78.5 cm²


Now we will find the diameter of circle circumscribed the square.
C

Diameter of circle circumscribed the square = diameter of square


©

= √(10)2 + (10)2
= 10√2 cm
So, radius of circle circumscribed the square = 5√2 cm
We know that the area 𝐴′ of circle inscribed the square is
2
𝐴′ = 𝜋𝑟 ′
Substituting the value of radius,
𝐴′ = 3.14 × 5√2 × 5√2
= 157 cm²
Hence the area of circle circumscribed the square is 157 cm² .
Page No 13.12:
Question 14: If a square is inscribed in a circle, find the ratio of the areas
of the circle and the square.
ANSWER:
Let ABCD be the square inscribed in a circle of radius r.

T IO S I
NO A D
CO N
A

PY
DO U A
Here, OA = OB = r.
T
∴ OA2 + OB2 = AB2
ED PR

⇒ r2 + r2 = AB2
C

⇒ 2r2 = AB2
©

Now, area of square ABCD = 𝐴𝐵2 = 2𝑟 2


Area of circle = 𝜋𝑟 2
Now we will find the ratio of area of the circle and the square.
Area of circle 𝜋𝑟 2 𝜋
= =
Area of square 2𝑟 2 2

Hence, the ratio of area of the circle and square is 𝜋: 2.

Page No 13.12:
Question 15: The area of a circle inscribed in an equilateral triangle is
154 cm2. Find the perimeter of the triangle. [Use π = 22/7 and √3 = 1.73]
ANSWER:
It is given that the area A of circle inscribed in an equilateral triangle is
154 cm2.
We know that the area A of circle inscribed in an equilateral triangle is
𝐴 = 𝜋𝑟 2
Now, we will find the value of r.
Substituting the value of area,

T IO S
154 = 3.14 × 𝑟 2

I
𝑟2 = NO A D
154

CO N
3.14
A
𝑟 2 = 49

PY
DO U A

𝑟 = √49
T
ED PR

𝑟 = 7 cm
Let the height of triangle be h. Then
C
©


𝑟=
3

ℎ = 3𝑟
=3×7
= 21 cm
If a is the side of triangle, then
√3
ℎ= 𝑎
2
2ℎ
𝑎=
√3

Substituting the value of h,


2×21
𝑎=
√3

= 14√3 cm
Perimeter of triangle = 3𝑎
= 3 × 14√3
= 42 × 1.732
= 72.7 cm

T IO S
Hence perimeter of triangle is 72.7 cm.

I
NO A D
CO N
A
Page No 13.12:

PY
DO U A
Question 16: A field is in the form of a circle. A fence is to be erected
T
around the field. The cost fencing would be Rs. 2640 at the rate of Rs. 12
ED PR

per metre. The, the field is to be thoroughly ploughed at the cost of Re.
0.50 per m2. What is the amount required to plough the field? [Take π =
C

22/7].
©

ANSWER:
We know that the circumference C of a circle of radius r is
𝐶 = 2𝜋𝑟
It is given that cost of fencing around the circular field would be Rs.2640
at the rate of Rs.12 per meter. So,
2𝜋𝑟 × 12 = 2650
22
24 × 𝑟 = 2650
7
2650×7
𝑟=
24×22

= 35 m
We know that the area A of circle of radius r,
𝐴 = 𝜋𝑟 2
Substituting the value of r
22
𝐴= × 35 × 35
7

= 3850 m²
Since, cost to plough per m² field = Rs. 0.50

T IO S
Then, cost to plough 3850 m² field = Rs. 0.50 × 3850

I
= Rs. 1925
NO A D
CO N
A
Hence, amount required to plough the field is Rs. 1925.

PY
DO U A
T
Page No 13.12:
ED PR

Question 17: A park is in the form of a rectangle 120 m × 100 m. At the


centre of the park there is a circular lawn. The area of park excluding lawn
C

is 8700 m2. Find the radius of the circular lawn. (Use π = 22/7).
©

ANSWER:
Let the radius of circular lawn be r. Then,
Area of circular lawn = 𝜋𝑟 2
It is given that
Area of park excluding lawn = Area of rectangle – Area of circular lawn
8700 = 120 × 100 − 𝜋𝑟 2
𝜋𝑟 2 = 12000 − 8700
22 2
𝑟 = 3300
7
3300×7
𝑟2 =
22
𝑟 2 = 1050
𝑟 = √1050
𝑟 = 32.40 m
Hence, radius of circular lawn is 32.40 m.

Page No 13.12:

T IO S
Question 18: A car travels 1 kilometre distance in which each wheel
makes 450 complete revolutions. Find the radius of the its wheels.

I
NO A D
ANSWER:

CO N
A
Let the radius of wheel be r. Thus, circumference C of the wheel

PY
DO U A
𝐶 = 2𝜋𝑟
T
ED PR

Since car travels 1 km distance in which wheel makes 450 complete


revolutions. Then
C

Distance moved
The distance covered by wheel in one revolution =
©

Number of revolution
1000 m
=
450
20
= m
9

We know that,
The distance covered in one revolution = circumference of the wheel
20 22
=2× ×𝑟
9 7
20×7
𝑟=
9×2×22

= 0.35 m
= 35.35 cm
Hence the radius of wheel is 35.35 cm.

Page No 13.12:
Question 19: The area enclosed between the concentric circles is 770 cm2.
If the radius of the outer circle is 21 cm, find the radius of the inner circle.
ANSWER:
Let the radius of outer and inner two circles be r1 and r2 respectively.

T IO S
Area enclosed between concentric circles = πr12 − πr22

I
22
NO A D
⇒ 770 = (212 − 𝑟22 )

CO N
7
A
⇒ 245 = 212 − 𝑟22

PY
DO U A
⇒ 𝑟22 = 441 − 245
T
ED PR

⇒ 𝑟22 = 196
⇒ 𝑟22 = 142
C

⇒ 𝑟 2 = 14 cm
©

Hence, the radius of inner circle is 14 cm.

Page No 13.12:
Question 20: An archery target has three regions formed by three
concentric circles as shown in figure15.8. If the diameters of the
concentric circles are in the ratios 1: 2: 3, then find the ratio of the areas
of three regions.
ANSWER:
T IO S
Let the three regions be A , B and C.

I
The diameters are in the ratio 1 : 2 : 3.
NO A D
CO N
Let the diameters be 1x, 2x and 3x
A
𝑥 2𝑥 3𝑥
Then the radius will be , and
2 2 2

PY
DO U A
𝑥 2 𝜋𝑥 2
Area of region A = 𝜋𝑟𝐴2 = 𝜋 ( ) =
T 2 4
ED PR

𝑥 2 3𝜋(𝑥)2
Area of region B = 𝜋𝑟𝐵2 − 𝜋𝑟𝐴2 = 𝜋(𝑥 )2 − 𝜋 ( ) =
2 4
3𝑥 2 𝑥 2
𝜋𝑟𝐶2 𝜋𝑟𝐵2 𝜋𝑟𝐴2 )2
C

Area of region C = − − = 𝜋 ( ) − 𝜋(𝑥 −𝜋( )


2 2
©

3𝑥 2 3𝜋𝑥 2 5𝜋𝑥 2
= 𝜋( ) − =
2 4 4
Thus, ratio of the areas of regions A, B and C will be
𝜋𝑥 2 3𝜋(𝑥)2 5𝜋𝑥 2
: :
4 4 4
⇒ 1: 3: 5

Page No 13.13:
Question 21: The wheel of a motor cycle is of radius 35 cm. How many
revolutions per minute must the wheel make so as to keep a speed of 66
km / hr?
ANSWER:
Radius of the wheel, r = 35 cm
circumference = 2𝜋𝑟 = 2𝜋 × 35 = 220 cm
Distance covered by the wheel in one revolution = 220 cm
Let the number of revolutions required be x.
So, distance covered by the wheel in x revolutions = 220x
Distance covered by the wheel in one minute = 220x
66×1000×100
The wheel covers = 110000 cm in one minute
60
So, 220𝑥 = 110000

T IO S
110000
⇒𝑥= = 500
220

I
Hence, 500 revolutions per min are needed.
NO A D
CO N
A
Page No 13.13:

PY
DO U A

Question 22: A circular pond is 17.5 m in diameter. It is surrounded by a


T
2m wide path, Find the cost of constructing the path at the rate of ₹25 per
ED PR

m2 .
C

ANSWER:
©

Diameter of the pond = 17.5 m


Radius of the pond = 8.75 m
Radius of the pond with the path = 8.75 + 2 = 10.75 m
Area of the path = Area of the pond along with the path − area of the pond
Area of the path = 𝜋[𝑅 2 − 𝑟 2 ]
= 𝜋[(10.75)2 − (8.75)2 ]
= 𝜋[(2)(19.5)]
= 122.46 m2
Cost of constructing the path = 25 × 122.46 = Rs 3061.5
Page No 13.13:
Question 23: A circular park is surrounded by a rod 21 m wide. If the
radius of the park is 105 m, find the area of the road.
ANSWER:
Radius of the park, r = 105 m
Radius of the park with the road, R = 105 + 21 = 126 m
Area of the road = Area of the park with the road − Area of the park
= 𝜋𝑅 2 − 𝜋𝑟 2

T IO S
= 𝜋[(126)2 − (105)2 ]

I
NO A D
= 𝜋(21)(231)

CO N
A
= 15246 m2

PY
DO U A
T
Page No 13.13:
ED PR

Question 24: A square of diagonal 8 cm is inscribed in a circle. Find the


area of the region lying outside the circle and inside the square.
C
©

ANSWER:
DISCLAIMER: There is some error in the given question.
We have solved the question by taking a square inscribed in a circle. Then
finding the area inside the circle and outside the square.
Diagonal of the square = 8 cm.
Let the side of the square be a cm.
In triangle BCD,
𝐵𝐶 2 + 𝐶𝐷2 = 𝐵𝐷 2
⇒ 𝑎 2 + 𝑎 2 = 82
⇒ 𝑎2 = 32 .....(i)
Radius of the circle will be R = 4 cm
Now area between the circle and the square will be

T IO S
Area of the circle−area of the square = 𝜋(𝑅 )2 − 𝑎2 = 𝜋(4)2 − 32 =

I
(16𝜋 − 32) cm2
NO A D
CO N
A
Page No 13.13:

PY
DO U A

Question 25: A path of 4 m width runs round a semi-circular grassy plot


T
ED PR

3
whose circumference is 163 m find.
7

ANSWER:
C
©

We have given AB = 4m and circumference of semicircle with radius OA


3
as 163 m.
7

We are asked to find the area between the two semi-circles.


For that we will first find OA.
3
𝜋𝑟 = 163
7
22
Now we will substitute 𝜋 =
7
22 3
× 𝑟 = 163
7 7
1144 7
⇒𝑟= ×
7 22

⇒ 𝑟 = 52
⇒ 𝑂𝐴 = 52 m
Now we will find OB.

T IO S I
∴ 𝑂𝐵 = 𝐴𝑂 + 𝐴𝐵
NO A D
CO N
∴ 𝑂𝐵 = 52 + 4
A
∴ 𝑂𝐵 = 56 m

PY
DO U A
Now we will find the area between two semi-circles as given below,
T
𝜋×56×56 𝜋×52×52
ED PR

∴ Area = −
2 2
𝜋
= (1568 − 1352) m2
C

2
©

𝜋
= × 216
2
22
= × 216
7×2

= 339.43 m2
Therefore, area of the path is 339.43 m2.
Now we will find the cost of gravelling the path.
Cost = 339.43 × 1.50
= Rs 509.14
Therefore, cost of gravelling the path is Rs 509.14.
Now we will find the cost of turfing the plot. For that we will find the area
of the plot.
1
Area of the plot= 𝜋𝑟 2
2
1 22
= × (52)2
2 7
1 22
= × × 52 × 52
2 7

= 4249.14

T IO S
Cost of turfing the plot = 4249.14 × 0.45

I
NO A D = Rs 1912.11

CO N
A
1
Area of the plot= 𝜋𝑟 2
2

PY
DO U A
1 22
= × (52)2
T
2 7
ED PR

1 22
= × × 52 × 52
2 7
C

= 4249.14
©

Cost of turfing the plot = 4249.14 × 0.45


= Rs 1912.11
Therefore, cost of the turfing the plot is Rs 1912.11.
Disclaimer: Due to some error in the question, we get the different
answers.

Page No 13.13:
Question 26: Find the area enclosed between two concentric circles of
radii 3.5 cm and 7 cm. A third concentric circle is drawn outside the 7 cm
circle, such that the area enclosed between it and the 7 cm circle is same
as that between the two inner circles. Find the radius of the third circle
correct to one decimal place.
ANSWER:

T IO S I
NO A D
CO N
A

PY
DO U A
The area enclosed between the two circles of radii 3.5 cm and 7 cm
= 𝜋(72 − 3.52 )
T
ED PR

= 115.5 cm2
Let the radius of the outermost circle be r cm.
C

Area between the circles with radius r and 7 cm=Area between the circles
©

with radius 7 cm and 3.5 cm


𝜋(𝑟 2 − 72 ) = 115.5
115.5
⇒ ( 𝑟 2 − 72 ) =
𝜋

⇒ 𝑟 2 = 36.75 + 49 = 85.75 cm2


⇒ 𝑟 = 9.26 cm

Page No 13.13:
Question 27: A path of width 3.5 m runs around a semi-circular grassy
22
plot whose perimeter is 72 m. Find the area of the path. (Use 𝜋 = )
7
ANSWER:

T IO S
Let the radius of the semicircular plot be r.

I
NO A D
Perimeter of the semi-circular grassy plot = 𝜋𝑟 + 2𝑟 = 72

CO N
A
⇒ 𝑟 = 14 cm

PY
Given that the width of the plot = 3.5 m
DO U A

Thus, the outer radius = 3.5 + 14 = 17.5 m


T
ED PR

𝜋𝑅 2 𝜋𝑟 2
Area of the path = −
2 2
C

𝜋
= (𝑅 2 − 𝑟 2 )
©

2
𝜋
= ((17.5)2 − (14)2 )
2

= 173.25 m2

Page No 13.13:
Question 28: A circular pond is of diameter 17.5 m. It is surrounded by a
2m wide path. Find the cost of constructing the path at the rate of ₹25 per
22
square metre (Use 𝜋 = )
7

ANSWER:
Diameter, d = 17.5 m
17.5
Radius, r = m
2
17.5
Radius of the pond with the 2 m wide path = 2 + m
2
Area of the circular path = Area of the pond with the path − area of the
pond
17.5 2 17.5 2
= 𝜋 (2 + ) −𝜋( )
2 2

= 122.57 m2
Cost of constructing the path = 25 × 122.57 = Rs 3064.2

T IO S I
NO A D Page No 13.13:

CO N
A
Question 29: The outer circumference of a circular race-track is 528 m.
The track is everywhere 14 m wide. Calculate the cost of levelling the

PY
DO U A
22
track at the rate of 50 paise per square metre. (Use 𝜋 = )
T 7
ED PR

ANSWER:
Let the radius of the inner circle and the race track be R m.
C

Outer circumference of the race track = 528 m


©

⇒ 2𝜋𝑅 = 528
528
⇒𝑅= = 84 m
2𝜋

Total radius of the outer circle = 84 − 14 = 70 m


Area of the circular track = Area of the outer circle − area of inner circle
= 𝜋(84)2 − 𝜋(70)2
= 𝜋[(84 − 70)(84 + 70)]
= 𝜋(14)(154)
= 6776 m2
Cost of levelling the track = 0.5 × 6776 = Rs 33880.
Page No 13.13:
Question 30:
A road which is 7 m wide surrounds a circular park whose
circumference is 352 m. Find the area of the road.
ANSWER:
Width of the road = 7 m
circumference of the circular park = 352 m

T IO S
⇒ 2𝜋𝑟 = 352

I
352
⇒𝑟= = 56 m
NO A D
2𝜋

CO N
Area of the road = Area of the circular park including the path − area of
A
the circular park

PY
= 𝜋(𝑟 + 7)2 − 𝜋𝑟 2
DO U A
T
= 𝜋[(56 + 7)2 − (56)2 ]
ED PR

= 𝜋[632 − 562 ]
C

= 𝜋[7 × 119]
©

= 2618 m2

Page No 13.13:
Question 31: Prove that the area of a circular path of uniform
width h surrounding a circular region of radius r is 𝜋ℎ(2𝑟 + ℎ).
ANSWER:
The width of the circular path = h
Let the inner circle be region A and the outer circle be region B

T IO S
Radius of region A = r

I
Radius of region B = r + h
NO A D
Area of the circular path = Area of region B − Area of region A

CO N
A
= 𝜋(𝑟 + ℎ)2 − 𝜋𝑟 2

PY
= 𝜋(𝑟 2 + ℎ2 + 2𝑟ℎ − 𝑟 2 )
DO U A
T
= 𝜋ℎ(ℎ + 2𝑟)
ED PR

Hence Proved
C
©
Exercise – 13.2
Page No 13.24:
Question 1: Find, in terms of π, the length of the arc that subtends an
angle of 30° at the centre of a circle of radius 4 cm.
ANSWER:
The arc length l of a sector of an angle θ in a circle of radius r is given by
𝜃
𝑙= × 2𝜋𝑟

T IO S
360°

I
It is given that 𝑟 = 4 cm and 𝜃 = 30°. Substituting the value of r and θ in
NO A D
above equation,

CO N
A
30°
𝑙= × 2𝜋 × 4 cm

PY
360°
DO U A
2𝜋
= cm
T
3
ED PR
C

Page No 13.24:
©

Question 2: Find the angle subtended at the centre of a circle of radius 5


cm by an arc of length (5π/3) cm.
ANSWER:
We know that the arc length l of a sector of an angle θ in a circle of
radius r is
𝜃
𝑙= × 2𝜋𝑟
360°
5𝜋
It is given that 𝑟 = 5 cm and length 𝑙 = cm. Substituting these value
3
in above equation,
5𝜋 𝜃
= × 2𝜋 × 5
3 360°
5𝜋 × 360° = 𝜃 × 2𝜋 × 5 × 3
𝜃 = 60°
Hence, the angle subtended at the centre of circle is 60°.

Page No 13.24:
Question 3: An arc of length 20π cm subtends an angle of 144° at the
centre of a circle. Find the radius of the circle.

T IO S
ANSWER:

I
NO A D
We know that the arc length l of a sector of an angle θ in a circle of

CO N
A
radius r is

PY
𝜃
DO U A
𝑙= × 2𝜋𝑟
360°
T
ED PR

It is given 𝑙 = 20𝜋 cm and angle 𝜃 = 144°.


Now we substitute the value of l and θ in above formula to find the value
C

of radius r of circle.
©

144°
20𝜋 cm = × 2𝜋𝑟
360°
20𝜋×360°
𝑟= cm
2𝜋×144°

𝑟 = 25 cm

Page No 13.24:
Question 4: An arc of length 15 cm subtends an angle of 45° at the centre
of a circle. Find in terms of π, the radius of the circle.
ANSWER:
We know that the arc length l of a sector of an angle θ in a circle of radius
r is
𝜃
𝑙= × 2𝜋𝑟
360°

It is given that 𝑙 = 15 cm and angle θ = 45°.


Now we substitute the value of l and θ in above formula to find the value
of radius r of circle.
45°
15 cm = × 2𝜋𝑟

T IO S
360°

I
15×360°
𝑟= cm
NO A D2𝜋×45°

CO N
A
60
𝑟= cm
𝜋

PY
DO U A
Page No 13.24:
T
ED PR

Question 5: Find the angle subtended at the centre of a circle of radius 'a'
by an arc of length (aπ/4) cm.
C

ANSWER:
©

We know that the arc length l of a sector of an angle θ in a circle of


radius r is
𝜃
𝑙= × 2𝜋𝑟
360°
𝑎𝜋
It is given 𝑙 = cm and radius 𝑟 = 𝑎 cm.
4

Now we substitute the value of l and r in above formula to find the value
of angle θ subtended at the centre of circle.
𝑎𝜋 𝜃
cm = × 2𝜋 × 𝑎
4 360°
𝑎𝜋×360°
𝜃=
2𝜋𝑎×4

𝜃 = 45°
Page No 13.24:
Question 6: A sector of a circle of radius 4 cm contains an angle of 30°.
Find the area of the sector.
ANSWER:
We know that the area A of a sector of an angle θ in the circle of radius r is
given by
𝜃
𝐴= × 𝜋𝑟 2
360°

T IO S
It is given that 𝑟 = 4 cm and angle 𝜃 = 30°.

I
NO A D
Now we substitute the value of r and θ in above formula,

CO N
A
30°
𝐴= × 𝜋 × 4 × 4 cm²

PY
360°
DO U A
4𝜋
= cm²
T
3
ED PR
C

Page No 13.24:
©

Question 7: A sector of a circle of radius 8 cm contains an angle of 135°.


Find the area of the sector.
ANSWER:
We know that the area A of a sector of an angle θ in the circle of radius r is
given by
𝜃
𝐴= × 𝜋𝑟 2
360°

It is given that 𝑟 = 8 cm and 𝜃 = 135°.


Now we substitute the value of r and θ in above formula,
135°
𝐴= × 𝜋 × 8 × 8 cm²
360°

= 24𝜋 cm²
Page No 13.25:
Question 8: The area of a sector of a circle of radius 2 cm is π cm2. Find
the angle contained by the sector.
ANSWER:
We know that the area A of a sector of an angle θ in the circle of radius r is
given by
𝜃
𝐴= × 𝜋𝑟 2
360°

T IO S
It is given that 𝑟 = 2 cm and area 𝐴 = 𝜋 cm².

I
NO A D
Now we substitute the value of r and A in above formula to find the value

CO N
A
of θ,

PY
𝜃
DO U A
𝜋= × 𝜋 × 2 × 2 cm²
360°
T
ED PR

360°×𝜋
𝜃=
𝜋×2×2

= 90°
C
©

Page No 13.25:
Question 9: The area of a sector of a circle of radius 5 cm is 5 π cm2. Find
the angle contained by the sector.
ANSWER:
We know that the area A of a sector of an angle θ in the circle of radius r is
given by
𝜃
𝐴= × 𝜋𝑟 2
360°

It is given that radius 𝑟 = 5 cm and area 𝐴 = 5𝜋 cm².


Now we substitute the value of r and A in above formula to find the value
of θ,
𝜃
5𝜋 = ×𝜋×5×5
360°
360°×5𝜋
𝜃=
𝜋×5×5

= 72°

T IO S
Page No 13.25:

I
NO A D
Question 10: Find the area of sector of the circle of radius 5 cm, if the

CO N
corresponding arc length is 3.5 cm.
A
ANSWER:

PY
DO U A

Radius, r = 5 cm
T
ED PR

Arc length, l = 3.5 cm


1
Area = 𝑙𝑟
2
C

1
⇒ 𝐴 = (3.5)(5) = 8.75 cm²
©

Page No 13.25:
Question 11: In a circle of radius 35 cm, an arc subtends an angle of 72°
at the centre. Find the length of the arc and area of the sector.
ANSWER:
We know that the arc length l and area A of a sector of an angle θ in the
𝜃 𝜃
circle of radius r is given by 𝑙 = × 2𝜋𝑟 and 𝐴= ×
360° 360°
𝜋𝑟 2 respectively.
It is given that, 𝑟 = 35 cm and 𝜃 = 72°.
We will calculate the arc length using the value of r and θ,
72°
𝑙= × 2𝜋 × 35 cm
360°
72° 22
= ×2× × 35 cm
360° 7

= 44 cm
Now, we will find the value of area A of the sector
72°
𝐴= × 𝜋 × 35 × 35 cm2
360°

= 770 cm2

T IO S I
NO A D
CO N
Page No 13.25:
A
Question 12: The perimeter of a sector of a circle of radius 5.7 m is 27.2

PY
DO U A
m. Find the area of the sector.
T
ED PR

ANSWER:
We know that the area A of a sector of circle of radius r and arc length l
C

is given by
©

1
𝐴 = 𝑙𝑟
2

Let OAB is the given sector. Then,


Perimeter of sector OAB = 27.2
OA + OB + arc AB = 27.2,
5.7 + 5.7 + arc AB = 27.2,
11.4 + arc AB = 27.2,
arc AB = 15.8 m
So, l = 15.8 m
Now substituting the value of r and l in above formula,

T IO S
1
𝐴 = × 15.8 × 5.7
2

I
NO A D
= 45.03 m2

CO N
A

PY
DO U A
Page No 13.25:
T
Question 13: The perimeter of a certain sector of a circle of radius 5.6 m
ED PR

is 27.2 m. Find the area of the sector.


ANSWER:
C
©

We know that the area A of a sector of circle of radius r and arc length l is
given by
1
𝐴 = 𝑙𝑟
2

Let OAB is the given sector. Then,


Perimeter of sector OAB = 27.2 m
OA + OB + arc AB = 27.2 m
5.6 + 5.6 + arc AB = 27.2 m
11.2 + arc AB = 27.2 m

arc AB = 16 m
So, l = 16 m
Now substituting the value of r and l in above formula,

T IO S
1
𝐴 = × 16 × 5.6

I
2

NO A D
= 44.8 m2

CO N
A

PY
DO U A
Page No 13.25:
T
ED PR

Question 14: A sector is cut-off from a circle of radius 21 cm. The angle
of the sector is 120°. Find the length of its arc and the area.
C

ANSWER:
©

We know that the arc length l and area A of a sector of circle at an


𝜃 𝜃
angle θ of radius r is given by 𝑙 = × 2𝜋𝑟 and angle 𝐴 = × 𝜋𝑟 2 .
360° 360°

Let OAB is the given sector.


It is given that 𝑂𝐴 = 21 cm and angle ∠𝐴𝑂𝐵 = 120°.
Now using the value of r and θ, we will find the value of l and A,
Arc length,
120° 22
𝑙= ×2× × 21 cm
360° 7

= 44 cm
Area of sector,
120° 22
𝑙= × × 21 × 21
360° 7

T IO S
= 462 cm2

I
NO A D
CO N
A
Page No 13.25:

PY
DO U A
Question 15: The minute hand of a clock is √21 cm long. Find the area
T
described by the minute hand on the face of the clock between 7.00 AM
ED PR

and 7.05 AM.


ANSWER:
C
©

We know that the area A of a sector of circle at an angle θ of radius r is


given by
𝜃
𝐴= × 𝜋𝑟 2 .
360°

We have,
Angle described by the minute hand in one minute = 6°l
So, angle described by the minute hand in five minute = 6° × 5 = 30°
Thus,
Area swept by the minute hand in 5 minute
= Area of a sector of angle 30° in the circle of radius √21 cm
30° 22
= × × √21 × √21 cm²
360° 7

= 5.5 cm²

Page No 13.25:
Question 16: The minute hand of a clock is 10 cm long. Find the area of
the face of the clock described by the minute hand between 8 AM and
8.25 AM.

T IO S
ANSWER:

I
NO A D
We know that the area A of a sector of circle at an angle θ of radius r is

CO N
A
given by

PY
𝜃
DO U A
𝐴= × 𝜋𝑟 2
360°
T
ED PR

We have,
Angle described by the minute hand in one minute = 6°
C

So, angle described by the minute hand in 25 minute = 6° × 25 = 150°


©

∴ Required area

150° 22
= × × (10)2
360° 7

= 130.95 cm2

Page No 13.25:
Question 17: A sector of 56° cut out from a circle contains area 4.4 cm2.
Find the radius of the circle.
ANSWER:
We know that the area A of a sector of circle at an angle θ of radius r is
given by
𝜃
𝐴= × 𝜋𝑟 2 .
360°

It is given that, Area of a sector = 4.4 cm² and angle 𝜃 = 56°.


We can find the value of r by substituting these values in above formula,
56° 22 2
𝐴= × 𝑟
360° 7

T IO S
56° 22 2
4.4 = × 𝑟

I
360° 7

𝑟2 = NO A D
360°
×
7
× 4.4

CO N
56° 22
A
𝑟2 = 9

PY
DO U A

𝑟 = √9
T
ED PR

𝑟 = 3 cm
C
©

Page No 13.25:
Question 18: Area of sector of central angle 2000 of a circle is 770 cm2.
Find the length of the corresponding arc of this sector.
ANSWER:
Area of the sector = 770 cm2
Central angle, 𝜃 = 200°
𝜃
Area of the sector = × Area of the circle
360
200
⇒ 770 = × 𝜋𝑟 2
360
⇒ 𝑟 ≈ 21
1
Now, Area of sector = 𝑙𝑟
2
1
⇒ 770 = 𝑙 × 21
2
220
⇒𝑙= cm
3

Page No 13.25:

T IO S
Question 19: The length of minute hand of a clock is 5 cm. Find the area

I
swept by the minute hand during the time period 6:05 am and 6:40 am.
NO A D
CO N
ANSWER:
A
Minute hand of the clock describes a circle of radius equal to its length

PY
DO U A
i.e. 5 cm.
T
ED PR

The minute hand rotates through 6o in one minute.


So, the minute hand will sweep 210o in one minute.
C

Area swept by the minute hand in one minute is the area of a sector of
©

angle 6o in a circle of radius 5 cm.


Area swept by the minute hand in 35 minutes is the area of a sector of
angle 210o in a circle of radius 5 cm
𝜃
Area of sector = × 𝜋𝑟 2
360
210
= × 𝜋 × (5 )2
360
5
= 45 cm2
6

Page No 13.25:
Question 20: The length of the minute hand of a clock is 14 cm. Find the
area swept by the minute hand in 5 minutes.
ANSWER:
Angle make by the minute hand in 1 minute = 6∘
Angle make by the minute hand in 5 minute = 5 ⨯ 6∘ = 30∘
Area of the sector having central angle is given by
30°
𝜋(14)2
360°
1 22
= × (14)2
12 7

T IO S
= 51.33 cm2

I
Hence, the area swept by minute hand in 5 minutes is 51.33 cm2
NO A D
CO N
A

PY
Page No 13.25:
DO U A
T
Question 21: In a circle of radius 21 cm, an arc subtends an angle of 60°
ED PR

at the centre. Find (i) the length of the arc (ii) area of the sector formed by
the arc. (Use π = 22/7)
C

ANSWER:
©

Here, we have θ = 60° and r = 21 cm

(i) The length of the arc is given by


60°
× 2𝜋(21)
360°
1 22
= ×2× × 21
6 7

= 22 cm

(ii) Area of the sector formed by the arc is given by


60°
𝜋(21)2
360°
1 22
= × (21)2
6 7
= 231 cm2

Page No 13.25:
Question 22: From a circular piece of cardboard of radius 3 cm two
sectors of 900 have been cutoff. Find the perimeter of the remaining
portion nearest hundredth centimeters (Take ππ = 22/ 7).
ANSWER:

T IO S
Since the sectors are of central angle 90° so, the sector is in the form of a

I
quadrant
NO A D
Two such quadrants are cut off. So, a semicircle is left.

CO N
A
Perimeter of the semicircle = πr
22

PY
= × (3)
DO U A
7
T
= 9.428 cm
ED PR
C

Page No 13.25:
©

Question 23: The area of sector is one-twelfth that of the complete circle.
Find the angle of the sector.
ANSWER:
𝜃
We know, 𝐴 = ×Area of the circle .....(i)
360°

Let the area of the circle be Ar.


1
Thus area of the sector = 𝐴𝑟 .....(ii)
12

From (i) and (ii) we have


1 𝜃
𝐴𝑟 = × 𝐴𝑟
12 360°
360°
⇒ =𝜃
12

⇒ 𝜃 = 30°

Page No 13.25:
Question 24: AB is a chord of a circle with centre O and radius 4
cm. AB is of length 4 cm. Find the area of the sector of the circle formed

T IO S
by chord AB.

I
ANSWER:
NO A D
CO N
We have to find the area of the sector AOB formed by the chord AB.
A

PY
DO U A
T
ED PR
C
©

We have OA = 4 cm and AB = 4 cm. So,


𝐴𝐵
𝐴𝐿 = cm
2
4
= cm
2

= 2 cm

Let ∠𝐴𝑂𝐵 = 2𝜃. Then,


∠𝐴𝑂𝐿 = ∠𝐵𝑂𝐿
=𝜃
In ∆OLA, we have
𝐴𝐿
sin 𝜃 =
𝑂𝐴
2
=
4
1
=
2
1
𝜃 = sin−1

T IO S
2

I
= 30°
NO A D
CO N
Hence, ∠𝐴𝑂𝐵 = 60°
A

PY
Now, using the value of ∠𝐴𝑂𝐵 and r we will find the area of sector AOB,
DO U A

𝜃
T
𝐴= × 𝜋𝑟 2
ED PR

360°
60°
= × 𝜋 × 4 × 4 cm²
360°
C

8𝜋
©

= cm²
3

Page No 13.25:
Question 25: In a circle of radius 6 cm, a chord of length 10 cm makes
an angle of 110° at the centre of the circle. Find:
(i) the circumference of the circle

(ii) the area of the circle

(iii) the length of the arc AB,

(iv) the area of the sector OAB.


ANSWER:
It is given that the radius of circle 𝑟 = 6 cm, length of chord = 10 cm and
angle at the centre of circle 𝜃 = 110°.
(i) We know that the Circumference C of circle of radius r is,

𝐶 = 2𝜋𝑟
22
=2× ×6
7

T IO S
264
=

I
7

NO A D
= 37.71 cm

CO N
A
(ii) We know that the Area A of circle of radius r is,

PY
DO U A
𝐴 = 𝜋𝑟 2
T
ED PR

22
= ×6×6
7
792
=
C

7
©

𝐴 = 113.1 cm²

(iii) We know that the arc length l of a sector of an angle θ in a circle of


radius r is
110° 22
𝑙= ×2× ×6
360° 7
110°
= × 37.68
360°

= 11.51 cm

(iv) We know that the area A of a sector of an angle θ in the circle of


radius r is given by
𝜃
𝐴= × 𝜋𝑟 2
360°
110° 22
= × ×6×6
360° 7
110°
= × 113.1
360°

= 34.5 cm²

T IO S
Page No 13.25:

I
Question 26: In the following figure, shows a sector of a circle, centre O,
NO A D
containing an angle 0°. Prove that:

CO N
A
𝜋𝜃
(i) Perimeter of the shaded region is (tan 𝜃 + sec 𝜃 + − 1)

PY
180
DO U A
𝑟2 𝜋𝜃
T
(ii) Area of the shaded region is (tan 𝜃 − )
ED PR

2 180
C
©

ANSWER:

It is given that the radius of circle is r and the angle ∠𝐴𝑂𝐶 = 𝜃°.
T IO S I
In ∆AOB,
NO A D
CO N
A
It is given that OA = r.

PY
𝑂𝐴
cos 𝜃 =
DO U A
𝑂𝐵
T
𝑂𝐴
ED PR

𝑂𝐵 =
cos 𝜃

𝑂𝐵 = 𝑟 sec 𝜃
C

𝐴𝐵
©

tan 𝜃 =
𝑂𝐴

𝐴𝐵 = 𝑂𝐴 tan 𝜃
𝐴𝐵 = 𝑟 tan 𝜃

(i) We know that the arc length l of a sector of an angle θ in a circle of


radius r is
𝜃
𝑙= × 2𝜋𝑟
360°

Perimeter of sector AOC = OC + OA + arc length AB


Now we substitute the value of OC, OA and l to find the perimeter of
sector AOC,
𝜃
Perimeter of sector AOC = 𝑟 + 𝑟 + × 2𝜋𝑟
360°
𝜃
= 2𝑟 + × 𝜋𝑟
180°

Perimeter of ∆AOB = OB + OA + AB
= 𝑟 sec 𝜃 + 𝑟 + 𝑟 tan 𝜃
= 𝑟(sec 𝜃 + tan 𝜃 + 1)
Perimeter of shaded region ABC

T IO S
= Perimeter of ∆AOB – Perimeter of sector AOC

I
NO A D = 𝑟(sec 𝜃 + tan 𝜃 + 1) − 2𝑟 −
𝜃
× 𝜋𝑟

CO N
180°
A
𝜋𝜃
= 𝑟 (sec 𝜃 + tan 𝜃 − − 1)

PY
180°
DO U A
T
ED PR

𝜋𝜃
Hence, Perimeter of shaded region ABC = 𝑟 (sec 𝜃 + tan 𝜃 − − 1).
180°

(ii) We know that area A of the sector at an angle θ in the circle of radius
C
©

r is
𝜃
𝐴= × 𝜋𝑟 2 .
360°

Thus
𝜃
Area of sector AOC = × 𝜋𝑟 2
360°
1
Area of ∆AOB = × 𝑂𝐴 × 𝐴𝐵
2
1
= × 𝑟 × 𝑟 tan 𝜃
2
1
= × 𝑟 2 tan 𝜃
2
Area of shaded region ABC = Area of ∆AOB – Area of sector AOC
1 𝜃
= × 𝑟 2 tan 𝜃 − × 𝜋𝑟 2
2 360°
𝑟2 𝜋𝜃
= (tan 𝜃 − )
2 360°
𝑟2 𝜋𝜃
Hence, Area of shaded region ABC = (tan 𝜃 − )
2 360°

Page No 13.26:

T IO S I
Question 27: Figure 15.18 shows a sector of a circle of radius r cm
NO A D
containing an angle θ°. The area of the sector is A cm2 and perimeter of

CO N
A
the sector is 50 cm. Prove that

PY
DO U A
360 25
(i) 𝜃 = ( − 1)
T
𝜋 𝑟
ED PR

(ii) A = 25r − r2
C
©

ANSWER:
It is given that the radius of circle is r cm and angle ∠𝐴𝑂𝐵 = 𝜃°.
(i) We know that the arc length l of a sector of an angle θ in a circle of
radius r is

T IO S
𝜃

I
𝑙= × 2𝜋𝑟
360°
NO A D
CO N
Perimeter of sector AOB = OB + OA + arc length AB
A
Now we substitute the value of OB, OA and l to find the perimeter of

PY
DO U A
sector AOB,
T
ED PR

𝜃
Perimeter of sector AOB = 𝑟 + 𝑟 + × 2𝜋𝑟
360°
𝜋𝜃
50 = 2𝑟 (1 + )
C

360°
©

25 𝜋𝜃
=1+
𝑟 360°
𝜋𝜃 25
= −1
360° 𝑟
360° 25
𝜃= ( − 1)
𝜋 𝑟

(ii) We know that area A of the sector at an angle θ in the circle of radius
r is
𝜃
𝐴= × 𝜋𝑟 2 .
360°

Thus
𝜃
Area of sector AOB = × 𝜋𝑟 2
360°

Substituting the value of θ,


360° 25
( 𝑟 −1)
A= 𝜋
𝜋𝑟 2
360°
25
𝐴=( − 1) 𝑟 2
𝑟

𝐴 = 25𝑟 − 𝑟 2

T IO S I
NO A D
CO N
A

PY
DO U A
T
ED PR
C
©
Exercise 13.3
Page No 13.32:
Question 1: AB is a chord of a circle with centre O and radius 4 cm. AB is
of length 4 cm and divides the circle into two segments. Find the area of
the minor segment.
ANSWER:
We know that the area of minor segment of angle θ in a circle of

T IO S
radius r is,

I
𝜋𝜃 𝜃 𝜃
𝐴={ − sin cos } 𝑟 2
NO A D
360° 2 2

CO N
A
It is given that the chord AB divides the circle in two segments.

PY
DO U A
T
ED PR
C
©

We have OA = 4 cm and AB = 4 cm. So,


𝐴𝐵
𝐴𝐿 = cm
2
4
= cm
2

= 2 cm
Let ∠𝐴𝑂𝐵 = 2𝜃. Then,
∠𝐴𝑂𝐿 = ∠𝐵𝑂𝐿
=𝜃
In ∆OLA, we have
𝐴𝐿
sin 𝜃 =
𝑂𝐴
2
=
4
1
=
2
1
𝜃 = sin−1
2

= 30°

T IO S
Hence, ∠𝐴𝑂𝐵 = 60°

I
Now using the value of r and θ, we will find the area of minor segment
NO A D
CO N
𝜋×60° 60° 60°
A
𝐴={ − sin cos }×4×4
360° 2 2

PY
DO U A
𝜋
= { − sin 30° cos 30°} × 16
6
T
ED PR

16×𝜋 1 √3
={ − × × 16}
6 2 2
C

8𝜋
={ − 4√3} cm²
3
©

Page No 13.32:
Question 2: A chord PQ of length 12 cm subtends an angle of 120° at the
centre of a circle. Find the area of the minor segment cut off by the
chord PQ.
ANSWER:
We know that the area of minor segment of angle θ in a circle of
radius r is,
𝜋𝜃 𝜃 𝜃
𝐴={ − sin cos } 𝑟 2
360° 2 2

It is given that the chord PQ divides the circle in two segments.


We have ∠𝑃𝑂𝑄 = 120° and 𝑃𝑄 = 12 cm. So,
𝑃𝑄

T IO S
𝑃𝐿 = cm
2

I
12
=
NO A D
2
cm

CO N
A
= 6 cm

PY
DO U A
Since ∠𝑃𝑂𝑄 = 120°,
T
∠𝑃𝑂𝐿 = ∠𝑄𝑂𝐿
ED PR

= 60°
C

In ∆OPQ, we have
©

𝑃𝐿
sin 𝜃 =
𝑂𝐴
6
sin 60° =
𝑂𝐴

√3 6
=
2 𝑂𝐴
12
𝑂𝐴 =
√3

Thus the radius of circle is OA = 4√3 cm.


Now using the value of radius r and angle θ we will find the area of minor
segment
120°𝜋 120° 120° 2
𝐴={ − sin cos } (4√3)
360° 2 2
𝜋 √3 1
={ − × } × 48
3 2 2

= 4{4𝜋 − 3√3} cm²

Page No 13.32:
Question 3: A chord of a circle of radius 14 cm makes a right angle at the
centre. Find the areas of the minor and major segments of the circle.

T IO S
ANSWER:

I
We know that the area of minor segment of angle θ in a circle of
NO A D
CO N
radius r is,
A
𝜋𝜃 𝜃 𝜃
𝐴={ − sin cos } 𝑟 2

PY
DO U A
360° 2 2
T
It is given that the chord of the circle of radius 𝑟 = 14 cm makes right
ED PR

angle at the centre.


So, 𝜃 = 90°
C

Substituting the value of r and angle θ in above formula,


©

Area of minor segment


90°𝜋 90° 90°
𝐴={ − sin cos } × 14 × 14
360° 2 2
𝜋
= { − sin 45° cos 45°} × 196
4
22×196 1 1
= − × × 196
7×4 √2 √2

= 154 − 98
= 56 cm²
Hence, area of minor segment is 56 cm²
Area of circle = 𝜋𝑟 2
22
= × 14 × 14
7

= 616 cm²
Area of major segment = Area of circle – Area of minor segment
= 616 – 56
= 560 cm²

T IO S I
Page No 13.32:
NO A D
CO N
A
Question 4: A chord 10 cm long is drawn in a circle whose radius is
5√2 cm. Find area of both the segments.

PY
DO U A

ANSWER:
T
ED PR

We know that the area of minor segment of angle θ in a circle of


radius r is,
C

𝜋𝜃 𝜃 𝜃
𝐴={ − sin cos } 𝑟 2
©

360° 2 2

It is given that the chord AB divides the circle in two segments.

We have OA = 5√2 and AB = 10 cm. So,


𝐴𝐵
𝐴𝐿 = cm
2
10
= cm
2

= 5 cm
Let ∠𝐴𝑂𝐵 = 2𝜃. Then,
∠𝐴𝑂𝐿 = ∠𝐵𝑂𝐿
=𝜃
In ∆OLA, we have

T IO S
𝐴𝐿
sin 𝜃 =

I
𝑂𝐴

NO A D
=
5

CO N
5√2
A
1
=

PY
DO U A
√2
T 1
𝜃 = sin−1
ED PR

√2

= 45°
C

Hence, ∠𝐴𝑂𝐵 = 90°


©

Now using the value of r and θ, we will find the area of minor segment
90°𝜋 90° 90°
𝐴={ − sin cos } × 5√2 × 5√2
360° 2 2
𝜋
= { − sin 45° cos 45°} × 50
6
3.14×50 1 1
= − × × 50
4 √2 √2

= 39.25 − 25
𝐴 = 14.25 cm²
Area of circle = 𝜋𝑟 2
= 3.14 × 5√2 × 5√2
= 157.15 cm²
Area of major segment = Area of circle – Area of minor segment
= 157 – 14.25
= 142.75 cm²

Page No 13.32:
Question 5: A chord AB of a circle, of radius 14 cm makes an angle of

T IO S
60° at the centre of the circle. Find the area of the minor segment of the

I
circle. (Use π = 22/7)
NO A D
CO N
A
ANSWER:

PY
We know that the area of minor segment of angle θ in a circle of
DO U A

radius r is,
T
ED PR

𝜋𝜃 𝜃 𝜃
𝐴={ − sin cos } 𝑟 2
360° 2 2
C

It is given that,
©

𝑟 = 14 cm
𝜃 = 60°
Substituting these values in above formula
3.14×60° 60° 60°
𝐴={ − sin cos } × 14 × 14
360° 2 2
3.14
={ − sin 30° cos 30°} × 196
6
3.14×196 1 √3
= − × × 196
6 2 2

= 102.573 − 84.868
𝐴 = 17.70 cm²
Page No 13.32:
Question 6: Find the area of minor segment of a circle of radius 14 cm,
when the angle of the corresponding sector is 60°.
ANSWER:
𝜃 𝜃 𝜃
Area of the minor segment of the circle = × 𝜋𝑟 2 − 𝑟 2 sin cos
360 2 2
60 60 60
= × 𝜋(14)2 − (14)2 sin cos
360 2 2

T IO S
1 1 √3
= (14)2 [ 𝜋 − × ]

I
6 2 2

NO A D
308

CO N
=( − 49√3) cm2
A
3

PY
DO U A
Page No 13.32:
T
ED PR

Question 7: A chord of a circle of radius 20 cm sub tends an angle of


900 at the centre. Find the area of the corresponding major segment of the
circle (Use π = 3.14)
C
©

ANSWER:
𝜋𝜃 𝜃 𝜃
We know area of minor segment of the circle is 𝐴 = { − sin cos 𝑟 2
360 2 2
𝜋×90° 90 90
⇒𝐴={ − sin cos } (20)2
360 2 2
𝜋 1
⇒ 𝐴 = ( − ) (400)
4 2
Area of the major segment = Area of the circle − area of the minor
segment
𝜋 1
= 𝜋(20)2 − (400) [ − ]
2 2
𝜋 1
= (400) [𝜋 − + ]
2 2
= 1142 cm2

DISCLAIMER: The answer given in the book is incorrect. If we take the


radius 10 instead of 20, then the answer will match.

Page No 13.32:
Question 8: The radius of a circle with centre is 5 cm (Fig. 13.31). two
radii OA and OB are drawn at right angles to each other. Find the areas of

T IO S
the segments made by the chord AB (Take 𝜋 = 3.14)

I
NO A D
CO N
A

PY
DO U A
T
ED PR
C
©

ANSWER:

𝜋𝜃 𝜃 𝜃
We know area of minor segment of the circle is A= { − sin cos } 𝑟 2
360 2 2
𝜋90 90 90
𝐴={ − sin cos } (5)2
360 2 2
𝜋
𝐴 = { − sin 45 × cos 45} (5)2
4
𝜋 1 1
𝐴={ − × } 25
4 √2 √2

𝜋 1
𝐴 = { − } 25 = 7.125 cm²
4 2

Area of the major segment = Area of the sector − area of minor segment
= 𝜋(5)2 − 7.125
= 71.375 cm2

T IO S I
Page No 13.33:
NO A D
CO N
Question 9: AB is the diameter of a circle, centre O, C is a point on the
A
circumference such that ∠COB = θ. The area of the minor segment cut off

PY
DO U A
by AC is equal to twice the area of the sector BOC. Prove that
T
ED PR

𝜃 𝜃 1 𝜃
sin cos = 𝜋 ( − )
2 2 2 120
C
©

ANSWER:
We know that the area of minor segment of angle θ in a circle of
radius r is,

𝜋𝜃 𝜃 𝜃
𝐴={ − sin cos } 𝑟 2 .
360° 2 2
It is given that, ∠𝐵𝑂𝐶 = 𝜃
So, ∠𝐴𝑂𝐶 = 180° − 𝜃

T IO S
Area, A of minor segment cutoff by AC at angle ∠𝐴𝑂𝐶 = 180° − 𝜃

I
𝐴={ NO A D
𝜋(180°−𝜃)
− sin
(180°−𝜃)
cos
(180°−𝜃)
} 𝑟2

CO N
360° 2 2
A
Now, since sin(90° − 𝛼 ) = sin 𝛼 and cos(90° − 𝛼 ) = cos 𝛼

PY
DO U A
𝜋(180°−𝜃) 𝜃 𝜃
𝐴={ − sin cos } 𝑟 2
T
360° 2 2
ED PR

We know that the area of sector of a circle of radius r at an angle θ is


𝜃
𝐴′ = × 𝜋𝑟 2
C

360°
©

𝜃
So, the area of sector BOC, 𝐴′ = × 𝜋𝑟 2
360°

It is given that,
Area of minor segment cutoff by AC = 2 × Area of sector BOC
𝜋(180°−𝜃) 𝜃 𝜃 2𝜃
{ − cos sin } 𝑟 2 = × 𝜋𝑟 2
360° 2 2 360°
𝜋(180°−𝜃) 𝜃 𝜃 2𝜋𝜃
− cos sin =
360° 2 2 360°
𝜋180° 𝜋𝜃 𝜃 𝜃 2𝜋𝜃
− − cos sin =
360° 360° 2 2 360°
𝜃 𝜃 𝜋180° 𝜋𝜃 2𝜋𝜃
cos sin = − −
2 2 360° 360° 360°
𝜃 𝜃 𝜋 3𝜋𝜃
cos sin = −
2 2 2 360°
𝜃 𝜃 𝜋 𝜋𝜃
cos sin = −
2 2 2 120°
𝜃 𝜃 1 𝜃
cos sin = 𝜋 ( − )
2 2 2 120°

Page No 13.33:
Question 10: A chord of a circle subtends an angle of θ at the centre of

T IO S
the circle. The area of the minor segment cut off by the chord is one eighth

I
NO A D
of the area of the circle. Prove that

CO N
A
𝜃 𝜃 𝜋𝜃
8 sin cos + 𝜋 = .
2 2 45

PY
DO U A
ANSWER:
T
ED PR

We know that the area of circle and area of minor segment of angle θ in a
𝜋𝜃 𝜃 𝜃
circle of radius r is given by, 𝐴′ = 𝜋𝑟 2 and 𝐴 = { − sin cos } 𝑟 2
360° 2 2
C

respectively.
©

It is given that,
1
Area of minor segment = ×area of circle
8
𝜋𝜃 𝜃 𝜃 2 𝜋𝑟 2
{ − sin cos } 𝑟 =
360° 2 2 8
𝜋𝜃 𝜃 𝜃
{ − sin cos } × 8 = 𝜋
360° 2 2
8𝜋𝜃 𝜃 𝜃
− 8 sin cos = 𝜋
360° 2 2
𝜋𝜃 𝜃 𝜃
− 8 sin cos = 𝜋
45° 2 2
𝜋𝜃 𝜃 𝜃
= 8 sin cos + 𝜋
45° 2 2
Exercise 13.4
Page No 13.56:
Question 1: A plot is in the form of a rectangle ABCD having semi-circle
on BC as shown in the following figure. If AB = 60 m and BC = 28 m, find
the area of the plot.

T IO S I
NO A D
CO N
A
ANSWER:

PY
DO U A
It is given that a plot is in form of rectangle ABCD having a semicircle
T
on BC.
ED PR

AB = 60 m
C

BC = 28 m
©

Since BC is the diameter of semicircle. Then, radius of semicircle is


28
𝑟= m
2

= 14 m
1
Area of semicircle = × 𝜋𝑟 2
2
1 22
= × × 14 × 14
2 7
=308 m2
Area of rectangle ABCD = 𝑙 × 𝑏
= 60 × 28
= 1680 m2
Now,
Area of plot = Area of rectangle + Area of semicircle

T IO S
= 1680 + 308 m2

I
= 1988 m2
NO A D
CO N
A
Page No 13.56:

PY
DO U A
Question 2: A play ground has the shape of a rectangle, with two semi-
T
circles on its smaller sides as diameters, added to its outside. If the sides
ED PR

of the rectangle are 36 m and 24.5 m, find the area of the playground.
(Take π = 22.7).
C
©

ANSWER:
It is given that a playground has a shape of rectangle, with two semicircles
on its smaller sides as diameter, added to its outside. So,
Area of playground = Area of rectangle + 2 × Area of semicircle
We have, sides of rectangle l = 36 m and b = 24.5 m.
Since, the diameter of semicircle is 2𝑟 = 𝑏. Then,
24.5
𝑟=
2

= 12.25 m
𝜋𝑟 2
Area of semicircle =
2
1 22
= × × 12.25 × 12.25
2 7
= 235.81 m2
Area of rectangle ABCD = 𝑙 × 𝑏
= 36 × 24.5
= 882 m2
Thus, the area of playground is
Area of plot = Area of rectangle + Area of semicircle

T IO S
= 882 + 2 × 235.81 m2

I
NO A D = 882 + 471.62

CO N
= 1353.62 m2
A

PY
DO U A
T
Page No 13.56:
ED PR

Question 3: Find the area of the circle in which a square of area 64 cm2 is
inscribed. [Use π = 3.14]
C
©

ANSWER:

We have given area of the square.


∴ side2 = 64
∴ side2 = 8
Now we will find the diameter of the square.
∴ diagonal = √2 × side
∴ diagonal = √2 × 8
∴ diagonal = 8√2
We know that diagonal of the square is same as the diameter of the circle.
∴ diagonal = 8√2
∴ radius = 4√2

T IO S
Now we will find the area of the circle as shown below.

I
NO A D
∴ area of the circle = π × r 2

CO N
A
∴ area of the circle = π × 4√2 × 4√2

PY
DO U A
∴ area of the circle = 3.14 × 16 × 2
T
∴ area of the circle = 3.14 × 32
ED PR

∴ area of the circle = 100.48


C

Therefore, area of the circle is 100.48 cm².


©

Page No 13.56:
Question 4: A rectangular piece is 20 m long and 15 m wide. From its
four corners, quadrants of radii 3.5 m have been cut. Find the area of the
remaining part.
ANSWER:
It is given that, the quadrants of radius r have been cut from the four
corners of a rectangular piece is of length 𝑙 = 20 m and width 𝑤 = 15m.
We have to find the area of remaining part.
We know that,
Area of rectangle = 𝑙 × 𝑤
= 20 × 15
= 300 m²
1
Area of quadrant = 𝜋𝑟 2
4
1 22
= × × 3.5 × 3.5
4 7

= 9.625 m²

T IO S
Now,

I
NO A D
Area of remaining part = Area of rectangle – 4 × Area of quadrant

CO N
A
= 300 − 4 × 9.625

PY
DO U A
= 300 − 38.5
T
= 261.5 m²
ED PR
C

Page No 13.56:
©

Question 5: In the following figure, PQRS is a square of side 4 cm. Find


the area of the shaded square.

ANSWER:
Area of the shaded region is equal to the area of the square minus area of
four sectors with the same radius minus area of the circle.
We know that area of the four sectors with radius is equal to area of one
circle.
∴ Area of the shaded region = 42 − 𝜋 × 12 − 𝜋 × 12
∴ Area of the shaded region = 42 − 2 × 𝜋 × 1
∴ Area of the shaded region = 16 − 2𝜋

T IO S
Therefore, area of the shaded region is (16 − 2𝜋) cm²

I
NO A D
CO N
A
Page No 13.56:

PY
Question 6: Four cows are tethered at four corners of a square plot of side
DO U A

50 m, so that they just cannot reach one another. What area will be left
T
ED PR

unglazed? (Fig. 13.74)


C
©

ANSWER:
It is given that four cows are tethered at four corner of square ABCD. We
have to find the area of plot that will left ungrazed.
Let the side of square is a.

T IO S
a = 25 + 25 m = 50 m

I
NO A D
CO N
Area of square = a2
A
= 50 × 50

PY
DO U A

= 2500 m2
T
ED PR

1
Area of quadrant inside square = 𝜋𝑟 2
4
1 22
C

= × × 25 × 25
4 7
©

= 491.07 m²
Area of shaded region = Area of square – 4 × Area of quadrant
= 2500 − 4 × 491.07
= 2500 − 1964.28
= 535.71 m²

Page No 13.56:
Question 7: A cow is tied with a rope of length 14 m at the corner of a
rectangular field of dimensions 20 m ×16 m, find the area of the field in
which the cow can graze.
ANSWER:

T IO S I
NO A D
Shaded portion shows the area that is grazed by the cow. It is in the form

CO N
of a quadrant of a circle with radius 14 m.
A
1

PY
Area grazed by the cow = 𝜋𝑟 2
DO U A
4
T
1
= 𝜋(14)2
ED PR

= 154 m2
C
©

Page No 13.56:
Question 8: A calf is tied with a rope of length 6 m at the corner of a
square grassy lawn of side 20 m. If the length of the rope is increased by
5.5 m, find the increase in area of the grassy lawn in which the calf can
graze.
ANSWER:
T IO S I
NO A D
The area grazed by the calf is in the form of a quadrant of a circle with

CO N
A
radius 6 m.

PY
1
Area grazed by the calf with rope 6 m= 𝜋(6)2
DO U A
4
T
= 28.28 m2
ED PR

When the rope length is increased then total rope length = 6 + 5.5 m =
C

11.5 m
©

1
Area covered by the calf for grazing with rope 11.5 m = π(11.5)2
4

= 103.91 m2
Hence, increase in area of grassy lawn that is grazed = 103.91 − 28.28 =
75.63 m2

Page No 13.56:
Question 9: A square water tank has its side equal to 40 m. There are four
semi-circular grassy plots all round it. Find the cost of turfing the plot at
Rs. 1.25 per square metre (Take π = 3.14).
ANSWER:
It is given that the side of square 𝑎 = 40 m.
Since four semicircular grassy plots rounds a square water tank. Then,
diameter of semicircular plot is 2𝑟 = 𝑎.
So, the radius of semicircle
𝑎
𝑟=
2
40
=
2

= 20 m

T IO S
1
Area of semicircular plot = 𝜋𝑟 2
2

I
NO A D 1
= × 3.14 × 20 × 20

CO N
2
A
= 628 m²

PY
DO U A
Now, the total area of plot is sum of area of four semicircular plots.
T
ED PR

Total Area of plot = 4 × Area of semicircle


= 4 × 628 m²
C

= 2512 m²
©

Since, the cost of turfing the plot per square meter = Rs. 1.25
So, the cost of turfing 2512 square meter plot = Rs. 1.25 × 2512
= Rs. 3140/-

Page No 13.56:
Question 10: A rectangular park is 100 m by 50 m. It is surrounding by
semi-circular flower beds all round. Find the cost of levelling the semi-
circular flower beds at 60 paise per square metre (use π = 3.14).
ANSWER:
Since four semicircular flower beds rounds the rectangular park. Then,
diameters of semicircular plots are 2𝑟1 = 𝑙 and 2𝑟2 = 𝑤
So, the radius of semicircle at larger side of rectangle
𝑙
𝑟1 =
2
100
=
2

= 50 m

T IO S
1
Area of semicircular plot at larger sides of rectangle = 𝜋𝑟 2
2

I
NO A D 1
= × 3.14 × 50 × 50

CO N
2
A
= 3925 m²

PY
DO U A
And the radius of semicircle at smaller side of rectangle
T
ED PR

𝑙
𝑟2 =
2
50
C

=
2
©

= 25 m
1
Area of semicircular plot at smaller sides of rectangle = 𝜋𝑟 2
2
1
= × 3.14 × 25 × 25
2

= 981.25 m²
Now, the total area of semicircular plot is sum of area of four semicircular
plots.
Total Area of plot = 2 × 3925 + 2 × 981.25
= 7850 + 1962.5 m²
= 9812.5 m²
Since, the cost of levelling semicircular flower bed per square meter = Rs.
0.60.
So, the cost of levelling 9812.5 square meter flower bed
= Rs. 0.60 × 9812.5
= Rs. 5887.50/-

Page No 13.56:

T IO S
Question 11: The inside perimeter of a running track (shown in the

I
NO A D
following figure) is 400 m. The length of each of the straight portion is 90

CO N
m and the ends are semi-circles. If the track is everywhere 14 m wide. find
A
the area of the track. Also find the length of the outer running track.

PY
DO U A
T
ED PR
C
©

ANSWER:
It is given that, length of each straight portion = 90 m and width of track
= 14 m.

We know that the circumference C of semicircle of radius be r is


𝐶 = 𝜋𝑟
The inside perimeter of running track is the sum of twice the length of
straight portion and circumferences of semicircles. So,
inside perimeter of running track = 400 m
2𝑙 + 2𝜋𝑟 = 400 m
22
⇒ 2 × 90 + 2 × × 𝑟 = 400 m
7
220×7
⇒𝑟= = 35 m
2×22
Thus, radius of inner semicircle is 35 m.

T IO S
Now,

I
radius of outer semi-circle r' = 35 + 14 = 49 m
NO A D
CO N
Area of running track
A
= 2 × Area of rectangle + 2 × Area of outer semi circle − 2 ×

PY
DO U A
Area of inner semicircle
𝜋(49)2 𝜋(35)2
T
= 2 × 90 × 14 + 2 × −2×
ED PR

2 2
= 2520 + 𝜋 × (49 + 35)(49 − 35)
22
= 2520 + × 84 × 14
C

7
©

= 2520 + 3696 = 6216 m2


Hence, the area of running track = 6216 m2
Now, length L of outer running track is
𝐿 = 2 × 𝑙 + 2𝜋𝑟′
= 2 × 90 + 2𝜋 × 49
22
= 180 + 2 × × 49
7
= 180 + 308 = 488 m
Hence, the length L of outer running track is 488 m

Page No 13.57:
Question 12: Find the area of the following figure, in square cm, correct
to one place of decimal. (Take π = 22/7).
ANSWER:
Let the area of square ABCD be A.

T IO S
It’s given that, 𝐴𝐵 = 10 cm

I
NO A D
So, 𝐴 = 10 × 10 cm2

CO N
A

PY
DO U A
T
ED PR
C
©

It is given that a semicircle is attached to one side of the square.


The diameter of semicircle = 10 cm
So, radius r of semicircle = 5 cm
We know that the area of semicircle of radius r is
1
𝐴′ = 𝜋𝑟 2
2

Substituting the value of r,


1 22
𝐴′ = × ×5×5
2 7

= 39.3 cm²
From the above figure it is seen that a right angle triangle is cutoff from
one side of square.
1
The area of right angle triangle = 𝑏ℎ
2
1
= ×8×6
2

= 24 cm²
Now, the area of above figure is,
A” = Area of square + Area of semicircle – Area of triangle
= 100 + 39.3 − 24

T IO S
= 115.3 cm²

I
NO A D
Hence area of given figure is 115.3 cm²

CO N
A

PY
DO U A
Page No 13.57:
T
Question 13: In the following figure, from a rectangular region ABCD
ED PR

with AB = 20 cm, a right triangle AED with AE = 9 cm and DE = 12 cm,


is cut off. On the other end, taking BC as diameter, a semicircle is added
C

22
on outside' the region. Find the area of the shaded region. [Use 𝜋 = ]
©

[CBSE 2014]

ANSWER:
In right triangle AED
AD2 = AE2 + DE2
= (9)2 + (12)2
= 81 + 144
= 225
∴ AD2 = 225
⇒ AD = 15 cm
We know that the opposite sides of a rectangle are equal
AD = BC = 15 cm
Area of the shaded region = Area of rectangle − Area of triangle AED +
Area of semicircle
1 1 BC 2
= AB × BC − × AE × DE + π ( )

T IO S
2 2 2

I
1 1 22 15 2
NO A D
= 20 × 15 − × 9 × 12 + × ×( )

CO N
2 2 7 2
A
= 300 − 54 + 88.3928

PY
DO U A
= 334.3928 cm2
T
Hence, the area of shaded region is 334.39 cm2
ED PR
C

Page No 13.57:
©

Question 14: From each of the two opposite corners of a square of side 8
cm, a quadrant of a circle of radius 1.4 cm is cut. Another circle of radius
4.2 cm is also cut from the centre as shown in the following figure. Find
the area of the remaining (Shaded) portion of the square. (Use π = 22/7)
ANSWER:
It is given that a circle of radius 4.2 cm and two quadrants of radius 1.4
cm are cut from a square of side 8 cm.

T IO S I
NO A D
CO N
A

PY
DO U A
Let the side of square be a. Then,
T
Area of square = 𝑎2
ED PR

=8×8
C

= 64 cm²
©

Since the radius of circle is 4.2 cm. So,


Area of circle = 𝜋𝑟 2
22
= × 4.2 × 4.2
7

= 55.44 cm2
Now area of quadrant of circle of radius 1.4 cm is,
1
Area of quadrant = 𝜋𝑟 2
4
1 22
= × × 1.4 × 1.4
4 7

= 1.54 cm²
Area of shaded region = Area of square − Area of circle – 2 ×
Area of quadrant
= 64 − 55.44 − 2 × 1.54
= 5.48 cm2

Page No 13.57:
Question 15: In the following figure, ABCD is a rectangle with AB = 14

T IO S
cm and BC = 7 cm. Taking DC, BC and AD as diameters, three semi-

I
circles are drawn as shown in the figure. Find the area of the shaded
NO A D
CO N
region.
A

PY
DO U A
T
ED PR
C

ANSWER:
©

Area of the shaded region can be calculated as shown below,


Area of the shaded region = Area of rectangle − area of the semi-circle
with diameter DC triangle + 2 area of two semicircles with diameters
AD and BC
𝜋×72 𝜋×3.52
∴ Area of the shaded region = 7 × 14 − +2×
2 2
𝜋×49
∴ Area of the shaded region = 98 − + 𝜋 × 12.25
2
22
Substituting 𝜋 = we get,
7
22
7
×49 22
∴ Area of the shaded region = 98 − + × 12.25
2 7
22×7
∴ Area of the shaded region = 98 − + 22 × 1.75
2

∴ Area of the shaded region = 98 − 77 + 22 × 1.75


∴ Area of the shaded region = 21 + 38.5
∴ Area of the shaded region = 59.5
Therefore, area of the shaded region is 59.5 cm².

T IO S
Page No 13.58:

I
Question 16: In the following figure, ABCD is a rectangle, having AB =
NO A D
20 cm and BC = 14 cm. Two sectors of 180° have been cut off. Calculate:

CO N
A

PY
(i) the area of the shaded region.
DO U A

(ii) the length of the boundary of the shaded region.


T
ED PR
C
©

ANSWER:
(i) We have given two semi-circles and a rectangle.
Area of the shaded region = Area of the rectangle − Area of the two
semicircles ……. (1)
1
∴ Area of the shaded region = 20 × 14 − 2 × × 𝜋 × 7 × 7
2
22
Substituting 𝜋 = we get,
7
1 22
∴ Area of the shaded region = 20 × 14 − 2 × × ×7×7
2 7

∴ Area of the shaded region = 20 × 14 − 22 × 7


∴ Area of the shaded region = 280 − 154
∴ Area of the shaded region = 126
Therefore, area of shaded region is 126 cm².
(ii) Now we will find length of the boundary of the shaded region.

T IO S
∴ Length of the boundary the shaded region = 2𝜋𝑟 + 𝐴𝐵 + 𝐷𝐶

I
22
NO A D
∴ Length of the boundary the shaded region = 2 × × 7 + 20 + 20

CO N
7
A
∴ Length of the boundary the shaded region = 2 × 22 + 40

PY
DO U A
∴ Length of the boundary the shaded region = 44 + 40
T
∴ Length of the boundary the shaded region = 84
ED PR

Therefore, length of the boundary of the shaded region is 84 cm.


C
©

Page No 13.58:
Question 17: In the following figure, the square ABCD is divided into
five equal parts, all having same area. The central part is circular and the
lines AE, GC, BF and HD lie along the diagonals AC and BD of the
square. If AB = 22 cm, find:
(i) the circumference of the central part.
(ii) the perimeter of the part ABEF.
T IO S I
ANSWER:
NO A D
CO N
A
We have a square ABCD.

PY
DO U A
T
ED PR
C
©

We have,
AB = 22 cm

(i)We have to find the perimeter of the triangle. We have a relation as,
1
Area of circular region = (Area of ABCD)
5

So,
1
𝜋𝑟 2 = (22)2
5
22
𝑟=
√5𝜋

= 5.56
So perimeter of the circular region,
= 2𝜋𝑟
22 22
= (2) ( )

T IO S
7 √5𝜋

I
= 34.88 cm
NO A D
CO N
A
(ii)We have to find the perimeter of ABEF. Let O be the centre of the

PY
DO U A
circular region.
T
ED PR

Use Pythagoras theorem to get,


2(AE + 𝑟)2 = 222
C

AE + 𝑟 = 15.56
©

AE = (15.56 − 5.56) cm
= 10 c,
Similarly,
BE = 10 cm
Now length of arc EF,
Perimeter of circular region
=
4
34.88
= cm
4

= 8.64 cm
So, perimeter of ABFE,
= AB + BF + EF + AF
= (22 + 10 + 8.64 +10) cm
= 50.64 cm

Page No 13.58:
Question 18: In the following figure find the area of the shaded region.
(Use π = 3.14)

T IO S I
NO A D
CO N
A

PY
DO U A
T
ED PR
C
©

ANSWER:
Area of shaded region = Area of square − Area of 4 semicircle having
diameter 4 cm − Area of square having side 4 cm

1
= (Side)2 − 4 × πr 2 − (side)2
2

4 2
= (14)2 − 2 × 3.14 × ( ) − 42
2

= 196 − 25.12 − 16
= 154.88 cm2

T IO S I
Hence, the area of shaded region is 154.88 cm2
NO A D
CO N
A
Page No 13.58:

PY
DO U A
Question 19: In the following figure, OACB is a quadrant of a circle with
T
centre O and radius 3.5 cm. If OD = 2 cm, find the area of the (i)
ED PR

quadrant OACB (ii) shaded region.


C
©

ANSWER:
It is given that OACB is a quadrant of circle with centre at O and radius
3.5 cm.
T IO S I
NO A D
(i) We know that the area of quadrant of circle of radius r is,

CO N
A
1
𝐴 = 𝜋𝑟 2
4

PY
DO U A
Substituting the value of radius 𝑟 = 3.5 cm,
T
1 22
ED PR

𝐴= × × 3.5 × 3.5
4 7

= 9.625 cm²
C
©

Hence, the area of OACB is 9.625 cm².

(ii) It is given that radius of quadrant of small circle is 2 cm.


Let the area of quadrant of small circle be 𝐴′ .
1
𝐴′ = 𝜋𝑟 2
4
1 22
= × ×2×2
4 7

= 3.14 cm²
It is clear from the above figure that area of shaded region is the difference
of larger quadrant and the smaller one. Hence,
Area of shaded region = 𝐴 − 𝐴′
= 9.625 − 3.14
= 6.485 cm²

Page No 13.58:
Question 20: In the following figure a square OABC is inscribed in a
quadrant OPBQ of a circle. If OA = 21 cm, find the area of the shaded
region.

T IO S I
NO A D
CO N
A

PY
DO U A
T
ED PR

ANSWER:
C

Construction: Join OB
©

In right triangle AOB


OB2 = OA2 + AB2
= 212 + 212
= 441 + 441
= 882
∴ OB2 = 882
Area of the shaded region = Area of quadrant OPBQ − Area of Square
OABC
1
= 𝜋(𝑂𝐵)2 − (𝑂𝐴)2
4
1 22
= × × 882 − 441
4 7

= 693 − 441

T IO S
= 252 cm2

I
NO A D
Hence, the area of the shaded region is 252 cm2.

CO N
A

PY
DO U A
Page No 13.59:
T
Question 21: In the following figure, OABC is a square of side 7 cm. If
ED PR

OAPC is a quadrant of a circle with centre O, then find the area of the
shaded region. (Use π = 22/7)
C
©

ANSWER:
Area of shaded region = Area of square OABC − Area of quadrant OAPC
1
= (Side)2 − πr 2
4
1 22
= (7)2 − × ×7×7
4 7
= 49 − 38.5
= 10.5 cm2
Hence, the area of the shaded region is 10.5 cm2

Page No 13.59:
Question 22: In the following figure, OE = 20 cm. In sector OSFT,
square OEFG is inscribed. Find the area of the shaded region.

T IO S I
NO A D
CO N
A

PY
DO U A
T
ED PR
C

ANSWER:
©

We have to find the area of the shaded portion. We have, 𝑂𝐸 = 20 cm and


OEFG is a square.

Use Pythagoras theorem to find OF as,


OF = (√(20)2 + (20)2 ) cm

= 20√2 cm
So, radius of the circle,
OF = 20√2 cm
Therefore, area of the shaded region,
Area of the shaded region = (Area of quadrant of circle) – (Area of square)

T IO S
So,

I
NO A D
Area of the shaded region =
π(OF)2
− (OE)2

CO N
4
A
2
22 (20√2)

PY
= ( )( ) − (20)2
DO U A
7 4
T
= 628 − 400
ED PR

= 228 cm²
C
©

Page No 13.59:
Question 23: Find the area of the shaded region in the following figure,
if AC = 24 cm, BC = 10 cm and O is the centre of the circle. (Use π =
3.14)
ANSWER:
It is given a triangle ABC is cut from a circle.
𝐴𝐶 = 24 cm
𝐵𝑐 = 10 cm
1
Area of ∆ABC = 𝐴𝐶 × 𝐵𝐶
2
1
= × 24 × 10
2

T IO S
= 120 cm²

I
NO A D
CO N
A

PY
DO U A
T
ED PR
C
©

In ∆ABC,
∠𝐴𝐶𝐵 = 90°, Since any angle inscribed in semicircle is always right
angle.
By applying Pythagoras theorem,
𝐴𝐵2 = 𝐴𝐶 2 + 𝐵𝐶 2
= 24 × 24 + 10 × 10
= 576 + 100
= 676 cm²
𝐴𝐵
𝑂𝐴 =
2
26
= cm
2

= 13 cm
We know that the area A of circle of radius r is
𝐴 = 𝜋𝑟 2
Substituting the value of radius r,

T IO S
𝐴 = 3.14 × 13 × 13

I
NO A D
= 530.66 cm²

CO N
A
1
Area of semicircle = 𝜋𝑟 2
2

PY
DO U A
530.66
= cm²
T
2
ED PR

= 265.33 cm²
Area of shaded region = Area of circle – Area of semicircle – Area of
C
©

triangle
= 530.66 − 265.33 − 120
= 145.33 cm²

Page No 13.59:
Question 24: A circle is inscribed in an equilateral triangle ABC is side
12 cm, touching its sides (the following figure). Find the radius of the
inscribed circle and the area of the shaded part.
T IO S I
ANSWER:
NO A D
CO N
We have to find the area of the shaded portion. We have ∆ABC which is
A
an equilateral triangle and AB = 12 cm.

PY
DO U A
T
ED PR
C
©

We have O as the in centre and OP, OQ and OR are equal.


So,
𝑎𝑟(∆𝐴𝐵𝐶 ) = 𝑎𝑟(∆O𝐴𝐵) + 𝑎𝑟(∆𝑂𝐵𝐶 ) + 𝑎𝑟(∆𝑂𝐶𝐴)
Thus,
√3 1
(12)2 = 3 ( (12)(𝑟))
4 2
36√3
𝑟= cm
18

= 2√3 cm
So area of the shaded region,
= 𝑎𝑟(∆𝐴𝐵𝐶 ) − Area of the circle
√3 22 2
= (12)2 − (2√3)
4 7

= (62.35 − 37.71) cm²

T IO S I
= 24.64 cm²
NO A D
CO N
A
Page No 13.59:

PY
DO U A
Question 25: In the following figure, an equilateral triangle ABC of side
T
ED PR

6 cm has been inscribed in a circle. Find the area of the shaded region.
(Take π = 3.14).
C
©

ANSWER:
We have to find the area of the shaded portion. We have ∆𝐴𝐵𝐶 which is
an equilateral triangle and 𝐴𝐵 = 6 cm. Let r be the radius of the circle.
We have O as the circumcentre.

T IO S
∠𝑂𝐵𝐴 = 30°

I
So,
NO A D
CO N
3
A
cos(30°) =
𝑟

PY
DO U A
Thus, 𝑟 = 2√3
T
ED PR

So area of the shaded region,


= Area of the circle – 𝑎𝑟(∆𝐴𝐵𝐶 )
C

2 √3
=(3.14)(2√3) − (6)2
©

= (37.68 − 15.59) cm²


= 22.126 cm²

Page No 13.59:
Question 26: A circular field has a perimeter of 650 m. A square plot
having its vertices on the circumference of the field is marked in the field.
Calculate the area of the square plot.
ANSWER:
We have a circular field in which a square field is marked.
Let the radius of the circle be r. We have,
Perimeter = 650

T IO S
2𝜋𝑟 = 650

I
NO A D
𝑟=
325

CO N
𝜋
A
Use Pythagoras theorem to find the side of square as,

PY
DO U A

AB = √𝑟 2 + 𝑟 2
T
ED PR

325
=
𝜋
√2
C

So area of the square plot,


©

= (AB)2
325 2
=( √2) m²
𝜋

325(7) 2
=( √2) m²
22

= 21387 m²
Page No 13.59:
Question 27: Find the area of a shaded region in the following figure,
where a circular arc of radius 7 cm has been drawn with vertex A of an
equilateral triangle ABC of side 14 cm as centre.
(Use π = 22/7 and √3 = 1.73)
T IO S I
ANSWER:
NO A D
CO N
In equilateral triangle all the angles are of 60°
A
∴ ∠BAC = 60°

PY
Area of the shaded region
DO U A
T
= (Area of triangle ABC − Area of sector having central angle 60°) +
ED PR

Area of sector having central angle (360° − 60°)


C

√3 60° 300°
= (AB)2 − 𝜋(7)2 + 𝜋(7)2
©

4 360° 360°

√3 1 22 5 22
= (14)2 − × (7)2 + × (7)2
4 6 7 6 7

= 84.77 − 25.67 + 128.35


= 187.45 cm2

Hence, the area of shaded region is 187.45 cm2

Page No 13.60:
Question 28: A regular hexagon is inscribed in a circle. If the area of
hexagon is 24√3 cm2, find the area of the circle. (Use π = 3.14)
ANSWER:
Let the radius of the circle be r and side of hexagon be a.
3√3
Area of hexagon = 𝑎2
2
3√3
⇒ 24√3 = 𝑎2
2

⇒ 𝑎2 = 16
⇒ 𝑎 = 4 cm

T IO S
In an regular hexagon inscribed in a circle, its side is equal the radius.
∴ r = a = 4 cm

I
NO A D
Now, Area of circle is given by

CO N
𝜋𝑟 2 = 3.14 × 4 × 4
A
= 50.24 cm2

PY
DO U A
T
ED PR

Page No 13.60:
Question 29: ABCDEF is a regular hexagon with centre O (in the
C

following figure). If the area of triangle OAB is 9 cm2, find the area of :
©

(i) the hexagon and (ii) the circle in which the hexagon is inscribed.

ANSWER:
We know that a regular hexagon is made up of 6 equilateral triangles.
We have given area of the one of the triangles.
∴ Area of the hexagon = 6 × area of one equilateral triangle
∴ Area of the hexagon = 6 × 9
∴ Area of the hexagon = 54
We know that if a regular hexagon is inscribed in the circle, then the radius
of the circle is same as the side of the regular hexagon.
We also know that a regular hexagon is made up of 6 equilateral triangles

T IO S
and we have area of one of the equilateral triangle.

I
NO A D
∴ Area of the hexagon =
√3
× side2

CO N
4
A
Substituting the value of the given equilateral triangle we get,

PY
DO U A
√3
∴9= × side2
T
4
ED PR

9×4
∴ side2 =
√3
C

36
∴ side2 =
©

√3

Now we will find the area of the circle.


∴ Area of the circle = 𝜋𝑟 2
Substituting the values, we get,
22 36
∴ Area of the circle = ×
7 √3

Now we will substitute √3 = 1.732 we get,


22 36
∴ Area of the circle = ×
7 1.732
792
∴ Area of the circle =
12.124

∴ Area of the circle = 65.324


Therefore, area of the hexagon and area of the circle are 54 cm²
and 65.324 cm² respectively.

Page No 13.60:
Question 30: Four equal circles, each of radius 5 cm, touch each other as
shown in the following figure. Find the area included between them
(Take π = 3.14).

T IO S I
NO A D
CO N
A

PY
DO U A
T
ED PR
C
©

ANSWER:
It is given that four equal circle touches each other as shown in figure.

Let the side of square is a.


𝑎 =5+5
= 10 cm
Area of square = 𝑎2
= 10 × 10
= 100 cm²
We know that
Area of circle of radius 𝑟 = 𝜋𝑟 2

T IO S I
= 3.14 × 5 × 5
NO A D
CO N
= 78.5 cm²
A
1
Area of quadrant inside square = 𝜋𝑟 2

PY
DO U A
4
T
= 14 × 78.5 cm2
ED PR

Area of shaded region = Area of square – 4 × Area of quadrant


1
= 100 − 4 × × 78.5
C

4
©

= 100 − 78.5
= 21.5 cm2

Page No 13.60:
Question 31: Four equal circles, each of radius a, touch each other. Show
6
that the area between them is 𝑎2 (Take π = 22/7).
7

ANSWER:
It is given that four equal circles of radius a touches each other.
T IO S
So, Area of circle = 𝜋𝑎2

I
NO A D
Since circles touches each other, the lines joining their centre make a

CO N
A
square ABCD.

PY
The side of square is 2a.
DO U A
T 1
Area of quadrant inside square = 𝜋𝑎2
ED PR

Area of shaded region = Area of square – 4 × Area of quadrant


C

𝜋𝑎2
= (2𝑎)2 − 4 ×
©

4
22
= 4𝑎2 − 𝑎2
7
6
= 𝑎2
7

Page No 13.60:
Question 32: A child makes a poster on a chart paper drawing a square
ABCD of side 14 cm. She draws four circles with centre A, B, C and D in
which she suggests different ways to save energy. The circles are drawn
in such a way that each circle touches externally two of the three
remaining circles (in the following figure). In the shaded region she writes
a message 'Save Energy'. Find the perimeter and area of the shaded region.
(Use π = 22/7)

T IO S
ANSWER:

I
NO A D
Perimeter of the shaded portion = 4 ⨯ Length of the arc having central

CO N
A
angle 90∘
90° 22 14

PY
=4× ×2× ×
DO U A
360° 7 2
T
1 22
ED PR

=4× ×2× ×7
4 7

= 44 cm
C
©

Area of shaded portion = Area of square ABCD − 4 ⨯ Area of the arc


having central angle 90∘
90° 22 14 2
= (14 )2 −4× × ×( )
360° 7 2
1 22
= 196 − 4 × × × (7)2
4 7

= 196 − 154
= 42 cm2
Page No 13.61:
Question 33: The diameter of a coin is 1 cm (in the following figure). If
four such coins be placed on a table so that the rim of each touches that of
the other two, find the area of the shaded region (Take π = 3.1416).
ANSWER:

T IO S
Look at the figure carefully shaded region is bounded between four

I
NO A D
sectors of the circle with same radius and a square of side 1 cm.

CO N
A
Therefore, the area of the shaded region is nothing but the difference the
area of the square and area of one circle.

PY
DO U A

∴ Area of the shaded region = Area of square – Area of a circle


T
ED PR

∴ Area of the shaded region = 12 − 𝜋(0.5)2


∴ Area of the shaded region = 1 − 0.25𝜋
C
©

Substituting 𝜋 = 3.1416 we get,


∴ Area of the shaded region = 1 − 3.1416 × 0.25
∴ Area of the shaded region = 1 − 0.7854
∴ Area of the shaded region = 0.2146
Therefore, area of the shaded region is 0.2146 cm².

Page No 13.61:
Question 34: Two circular pieces of equal radii and maximum area,
touching each other are cut out from a rectangular card board of
dimensions 14 cm × 7 cm. Find the area of the remaining card board.
(Use π = 22/7).
ANSWER:
We know that we can cut two circular pieces of equal radii and
maximum area from the rectangular cardboard whose diameter is equal
to the width of the rectangular cardboard.
∴ Radii of two circular pieces = Half of the width of the rectangular
cardboard = 3.5 cm
Now,

T IO S
Area of remaining cardboard = Area of rectangular cardboard − 2 ⨯

I
Area of circular piece having radius 3.5 cm
NO A D 22

CO N
= 14 × 7 − 2 ( × 3.5 × 3.5)
7
A
= 98 − 77

PY
DO U A

= 21 cm2
T
ED PR

Hence, the area of the remaining cardboard is 21 cm2


C
©

Page No 13.61:
Question 35: In the following figure, AB and CD are two diameters of a
circle perpendicular to each other and OD is the diameter of the smaller
circle. If OA = 7 cm, find the area of the shaded region.
ANSWER:
It is given that AB and CD are two diameters of a circle perpendicular to
each other and OD is the diameter of small circle.

T IO S I
NO A D
CO N
A

PY
DO U A
T
ED PR

It is given that, OA = 7 cm
So, radius r of small circle is
C

7
𝑟 = cm
©

= 3.5 cm
We know that the area A of circle of radius r is 𝐴 = 𝜋𝑟 2 .
Substituting the value of r in above formula,
22
𝐴= × 3.5 × 3.5
7

= 38.5 cm²
Now, let the area of large circle be 𝐴′ .
Using the value radius OA,
22
𝐴′ = ×7×7
7
= 154 cm²
Hence,
Area of shaded region = Area of large circle – Area of small circle
= 154 – 38.5
= 115.5 cm²

T IO S
Page No 13.61:

I
Question 36: In the following figure, PSR, RTQ and PAQ are three
NO A D
semicircles of diameter 10 cm, 3 cm and 7 cm respectively. Find the

CO N
A
perimeter of shaded region. [CBSE 2014]

PY
DO U A
T
ED PR
C
©

ANSWER:
Perimeter of shaded region = Length of the arc PAQ + Length of the arc
PSR + Length of the arc RTQ
1 1 1
= × 2𝜋𝑟1 + × 2𝜋𝑟2 + × 2𝜋𝑟3
2 2 2
1 7 1 10 1 3
= × 2𝜋 ( ) + × 2𝜋 ( ) + × 2𝜋 ( )
2 2 2 2 2 2
7 3
= 𝜋 + 5𝜋 + 𝜋
2 2

= 10𝜋
= 31.4 cm

Hence, the perimeter of shaded region is 31.4 cm.

Page No 13.61:
Question 37: In the following figure, two circles with centres A and B
touch each other at the point C. If AC = 8 cm and AB = 3 cm, find the area
of the shaded region.

T IO S I
NO A D
CO N
A

PY
DO U A
T
ED PR
C
©

ANSWER:
Area of the shaded region can be calculated as shown below,
Area of the shaded region = Area of circle with radius AC − area of
circle with radius BC
We have given radius of the outer circle that is 8 cm but we don’t know
the radius of the inner circle.
We can calculate the radius of the inner circle as shown below,
BC = AC – AB
∴ 𝐵𝐶 = 8 − 3
∴ 𝐵𝐶 = 5
∴ Area of the shaded region = 𝜋 × 8 × 8 − 𝜋 × 5 × 5
∴ Area of the shaded region = 𝜋 × 64 − 𝜋 × 25
∴ Area of the shaded region = 𝜋 × 39
22
Substituting 𝜋 = we get,
7
22
∴ Area of the shaded region = × 39
7

∴ Area of the shaded region = 122.57

T IO S
Therefore, area of the shaded region is 122.57 cm².

I
NO A D
CO N
A
Page No 13.62:

PY
DO U A
Question 38: In the following figure, ABCD is a square of side 2a, Find
T
the ratio between
ED PR

(i) the circumferences


C
©

(ii) the areas of the in circle and the circum-circle of the square.

ANSWER:
We have a square ABCD having 𝐴𝐵 = 2𝑎. From the given diagram we
can observe that,
Radius of in circle (𝑟1 ) = 𝑎
Radius of circumcircle (𝑟2 ) = √2𝑎
(i) We have to find the ratio of the circumferences of the two circles. So
the required ratio is,
Perimeter of inner circle
=
Perimeter of circumcircle
2𝜋(𝑎)
=
2𝜋(√2𝑎)

T IO S
1
=

I
√2
NO A D
CO N
A
(ii) We have to find the ratio of the areas of the two circles. So the

PY
DO U A
required ratio is,
T
Area of inner circle
ED PR

=
Area of circumcircle
𝜋(𝑎)2
=
C

2
𝜋(√2𝑎)
©

1
=
2

Page No 13.62:
Question 39: In the following figure, there are three semicircles, A, B and
C having diameter 3 cm each, and another semicircle E having a circle D
with diameter 4.5 cm are shown. Calculate:
(i) the area of the shaded region
(ii) the cost of painting the shaded region at the rate of 25 paise per cm2 ,
to the nearest rupee.
T IO S
ANSWER:

I
(i) Area of the shaded region can be calculated as shown below,
NO A D
CO N
Area of the shaded region = Area of the semi-circle with diameter of 9 cm
A
− area of 2 semi-circles with radius 3cm − area of the circle with centre D

PY
DO U A
+ area of semi-circle with radius 3 cm
T
∴ Area of the shaded region
ED PR

𝜋×4.5×4.5 𝜋×1.5×1.5 𝜋×1.5×1.5


= −2× − 𝜋 × 2.25 × 2.25 +
2 2 2
C

𝜋×4.5×4.5 𝜋×1.5×1.5
©

∴ Area of the shaded region = − − 𝜋 × 2.25 × 2.25


2 2
𝜋
∴ Area of the shaded region = (20.25 − 2.25) − 𝜋 × 5.0625
2
𝜋
∴ Area of the shaded region = (18) − 𝜋 × 5.0625
2

∴ Area of the shaded region = 9𝜋 − 𝜋 × 5.0625


∴ Area of the shaded region = 𝜋(9 − 5.0625)
∴ Area of the shaded region = 3.937𝜋
22
Substituting 𝜋 = we get,
7
22
∴ Area of the shaded region = 3.9375 ×
7

∴ Area of the shaded region = 12.375


Therefore, area of the shaded region is 12.375 cm².
Now we will find the cost of painting the shaded region at the rate of 25
paise per cm2.
∴ Cost = 12.375 × 25
∴ Cost = 309.375 paise
∴ Cost = Rs. 3
Therefore, cost of painting the shaded region to the nearest rupee is Rs. 3.

T IO S I
NO A D Page No 13.62:

CO N
A
Question 40: In the following figure, ABC is a right-angled triangle, ∠B =

PY
90°, AB = 28 cm and BC = 21 cm. With AC as diameter a semicircle is
DO U A

drawn and with BC as radius a quarter circle is drawn. Find the area of the
T
ED PR

shaded region correct to two decimal places.


C
©

ANSWER:
We have given two semi-circles and one circle.
Area of the shaded region = area of semicircle with diameter AC + area
of right angled triangle ABC − area of sector
First we will find the hypotenuse of right angled triangle ABC.
𝐴𝐶 2 = 𝐴𝐵2 + 𝐵𝐶 2
∴ 𝐴𝐶 2 = 282 + 212
∴ 𝐴𝐶 2 = 784 + 441
∴ 𝐴𝐶 2 = 1225
∴ 𝐴𝐶 = 35
𝜋×17.5×17.5 1 𝜃
∴ Area of the shaded region = 2
+ × 28 × 21 −
2 360
× 𝜋 × 21 × 21

T IO S
𝜋×17.5×17.5 1
∴ Area of the shaded region = + 14 × 21 − × 𝜋 × 21 × 21
2 4

I
NO A D
Substituting 𝜋 =
22
we get,

CO N
7
A
22
×17.5×17.5 1 22
∴ Area of the shaded region =

PY
7
+ 14 × 21 − × × 21 × 21
DO U A
2 4 7
T 962.5 1
∴ Area of the shaded region = + 14 × 21 − × 11 × 3 × 21
ED PR

2 2

∴ Area of the shaded region = 481.25 + 294 − 346.5


C

∴ Area of the shaded region = 428.75


©

Therefore, area of shaded region is 428.75 cm².

Page No 13.63:
Question 41: In the following figure, O is the centre of a circular arc
and AOB is a straight line. Find the perimeter and the area of the shaded
region correct to one decimal place. (Take π = 3.142)
ANSWER:
(i) Let us find the perimeter of the shaded region.

T IO S
∴ Perimeter = 𝜋 × 10 + 12 + 16

I
NO A D
∴ Perimeter = 3.142 × 10 + 28

CO N
A
∴ Perimeter = 31.42 + 28

PY
DO U A
∴ Perimeter = 59.42
T
ED PR

Therefore, perimeter of the shaded region is 59.4 cm.


Now we will find the area of the shaded region can be calculated as
C

shown below,
©

Area of the shaded region = Area of the semi-circle − area of the right
angled triangle
First, we will find the length of AB as shown below,
𝐴𝐵2 = 𝐴𝐶 2 + 𝐶𝐵2
∴ 𝐴𝐵2 = 122 + 162
∴ 𝐴𝐵2 = 144 + 256
∴ 𝐴𝐵2 = 400
∴ 𝐴𝐵 = 20
𝜋×10×10 1
∴ Area of the shaded region = − × 12 × 16
2 2

∴ Area of the shaded region = 𝜋 × 50 − 6 × 16


∴ Area of the shaded region = 𝜋 × 50 − 96
Substituting 𝜋 = 3.142 we get,
∴ Area of the shaded region = 3.142 × 50 − 96
∴ Area of the shaded region = 157.1 − 96
∴ Area of the shaded region = 61.1
Therefore, area of the shaded region is 61.1 cm².

T IO S I
Page No 13.63:
NO A D
CO N
Question 42: In the following figure, the boundary of the shaded region
A
consists of four semi-circular arcs, the smallest two being equal. If the

PY
diameter of the largest is 14 cm and of the smallest is 3.5 cm, find
DO U A
T
(i) the length of the boundary.
ED PR

(ii) the area of the shaded region.


C
©

ANSWER:
(i) We will first find the length of the boundary.
Length of the boundary perimeter of semi-circle with diameter AB +
boundary of semi-circle with diameter 7 cm
Length of the boundary = (𝜋 × 7 + 𝜋 × 3.5) + 𝜋 × 1.75 + 𝜋 × 1.75
Length of the boundary = 𝜋(7 + 3.5 + 1.75 + 1.75)
∴ Length of the boundary = 14𝜋
∴ Length of the boundary = 44
Therefore, length of the boundary is 44 cm.

T IO S
Now we will find the area of the shaded region as shown below,

I
NO A D
Area of the shaded region = Area of the semi-circle with AB as a diameter

CO N
A
− area of the semi-circle with radius AE − area of the semi-circle with
radius BC + area of the semi-circle with diameter 7 cm.

PY
DO U A
𝜋×7×7 𝜋×1.75×1.75 𝜋×1.75×1.75
T
∴ Area of the shaded region = − − +
ED PR

2 2 2
𝜋×3.5×3.5
2
C

49𝜋 12.25𝜋
∴ Area of the shaded region = − 3.0625𝜋 −
©

2 2
61.25𝜋
∴ Area of the shaded region = − 3.0625𝜋
2
61.25𝜋−6.125𝜋
∴ Area of the shaded region =
2
22 1
∴ Area of the shaded region = 55.125 × ×
7 2

∴ Area of the shaded region = 7.875 × 11


∴ Area of the shaded region = 86.625
Therefore, area of the shaded region is 86.625 cm².

Page No 13.63:
Question 43: In the following figure, AB = 36 cm and M is mid-point
of AB. Semi-circles are drawn on AB, AM and MB as diameters. A circle
with centre C touches all the three circles. Find the area of the shaded
region.

T IO S I
NO A D
CO N
A

PY
ANSWER:
DO U A
T
We have given two semi-circles and one circle.
ED PR

Area of the shaded region = area of semicircle with diameter AB − area


of two semicircles with diameters AM and MB - area of circle ……. (1)
C
©

Let us calculate the area of the semi-circle with AB as a diameter.


𝜋𝑟 2
Area of semi-circle with AB as a diameter =
2
36 2
𝜋( 2 )
∴ Area of semi-circle with AB as a diameter =
2
𝜋×182
∴ Area of semi-circle with AB as a diameter =
2

Now we will find the area of the semi-circle with AM as a diameter.


𝜋𝑟 2
Area of semi-circle with AM as a diameter =
2
18 2
𝜋( 2 )
∴ Area of semi-circle with AM as a diameter =
2
𝜋×92
∴ Area of semi-circle with AM as a diameter =
2

Area of the semi-circle with MB as a diameter is same as the area of the


semi-circle with diameter with AM as a diameter.
Now we will find the area of the circle with centre C.
Area of circle = 𝜋𝑟 2

T IO S
We know that radius of the circle is one sixth of AB.

I
NO A D
Area of circle = 𝜋 × 62

CO N
A
Now we will substitute all these values in equation (1).

PY
DO U A
𝜋×182 𝜋×92 𝜋×92
∴ Area of the shaded region = − − − 36𝜋
T
2 2 2
ED PR

𝜋×182
∴ Area of the shaded region = − 𝜋 × 92 − 36𝜋
2
C

𝜋×182
∴ Area of the shaded region = − 81𝜋 − 36𝜋
©

2
𝜋×182
∴ Area of the shaded region = − 117𝜋
2

∴ Area of the shaded region = (162 − 117)𝜋


∴ Area of the shaded region = 45𝜋
Therefore, area of shaded region is 45𝜋 cm².

Page No 13.63:
Question 44: In the following figure, ABC is a right angled triangle in
which ∠A = 90°, AB = 21 cm and AC = 28 cm. Semi-circles are described
on AB, BC and AC as diameters. Find the area of the shaded region.
T IO S
ANSWER:

I
NO A D
We have given three semi-circles and one right angled triangle.

CO N
A
∴ Area of shaded region = Area of semi-circle with AB as a diameter

PY
DO U A
+ Area of semi-circle with AC as a diameter
T
ED PR

+ Area of right angled ABC


– Area of semi-circle with BC as a diameter
C

…….. (1)
©

Let us calculate the area of the semi-circle with AB as a diameter.


𝜋𝑟 2
Area of semi-circle with AB as a diameter =
2
21 2
𝜋( 2 )
∴ Area of semi-circle with AB as a diameter =
2

𝜋 21 2
∴ Area of semi-circle with AB as a diameter = ( )
2 2

Now we will find the area of the semi-circle with AC as a diameter.


𝜋𝑟 2
Area of semi-circle with AC as a diameter =
2
28 2
𝜋( 2 )
∴ Area of semi-circle with AC as a diameter =
2
𝜋 28 2
∴ Area of semi-circle with AC as a diameter = ( )
2 2

Now we will find the length of BC.


In right angled triangle ABC, we will use Pythagoras theorem,
𝐵𝐶 2 = 𝐴𝐵2 + 𝐴𝐶 2
∴ 𝐵𝐶 2 = 212 + 282
∴ 𝐵𝐶 2 = 441 + 786

T IO S
∴ 𝐵𝐶 2 = 1225

I
NO A D
∴ 𝐵𝐶 = 35

CO N
A
Now we will calculate the area of the right angled triangle ABC.

PY
1
DO U A
𝐴(∆𝐴𝐵𝐶 ) = × 𝐴𝐵 × 𝐴𝐶
2
T
ED PR

1
∴ 𝐴(∆𝐴𝐵𝐶 ) = × 21 × 28
2

∴ 𝐴(∆𝐴𝐵𝐶 ) = 21 × 14
C
©

∴ 𝐴(∆𝐴𝐵𝐶 ) = 294
Now we will find the area of the semi-circle with BC as a diameter.
𝜋𝑟 2
Area of semi-circle with BC as a diameter =
2
35 2
𝜋( 2 )
∴ Area of semi-circle with BC as a diameter =
2

𝜋 35 2
∴ Area of semi-circle with BC as a diameter = ( )
2 2

Now we will substitute all these values in equation (1).


𝜋 21 2 𝜋 28 2 𝜋 35 2
∴ Area of shaded region = ( ) + ( ) + 294 − ( )
2 2 2 2 2 2
𝜋
∴ Area of shaded region = (212 + 282 − 352 ) + 294
8
𝜋
∴ Area of shaded region = (441 + 784 − 1225) + 294
8
𝜋
∴ Area of shaded region = (1225 − 1225) + 294
8

∴ Area of shaded region = 294


Therefore, area of shaded region is 294 cm².

T IO S
Page No 13.64:

I
NO A D
Question 45: In the following figure, shows the cross-section of railway

CO N
A
tunnel. The radius OA of the circular part is 2 m. If ∠AOB = 90°,

PY
calculate:
DO U A
T
(i) the height of the tunnel
ED PR

(ii) the perimeter of the cross-section


C
©

(iii) the area of the cross-section.

ANSWER:
We have a cross section of a railway tunnel. ∆𝑂𝐴𝐵 is a right angled
isosceles triangle, right angled at O. let OM be perpendicular to AB.
(i) We have to find the height of the tunnel. We have,
OA = 2 m
Use Pythagoras theorem in ∆𝑂𝐴𝐵 to get,
AB = (√22 + 22 ) m

= 2√2 m
Let the height of the tunnel be h. So,

T IO S
1
Area of ∆𝑂𝐴𝐵 = (2)(2)
2

I
1
NO A D
(2√2)(𝑂𝑀) = 2

CO N
2
A
Thus,

PY
DO U A
OM = √2 m
T
ED PR

Therefore,
C

ℎ = (2 + √2) m
©

(ii)
Perimeter of cross-section is,
= major arc AB + AB
3
= 2(𝜋)(2) ( ) + 2√2
4

= (3𝜋 + 2√2) m
(iii)
Area of cross section,
3
= (Area of circle) + Area of ∆𝑂𝐴𝐵
4
3 1
= (𝜋)(2)2 ( ) + (2)(2)
4 2

= (3𝜋 + 2) m2

Page No 13.64:
Question 46: In the following figure, shows a kite in which BCD is the
shape of a quadrant of a circle of radius 42 cm. ABCD is a square and
Δ CEF is an isosceles right angled triangle whose equal sides are 6 cm

T IO S
long. Find the area of the shaded region.

I
NO A D
CO N
A

PY
DO U A
T
ED PR
C
©

ANSWER:
We will find the area of the shaded region as shown below,
Area of the shaded region = area of quadrant + area of isosceles triangle
……. (1)
90 1
∴ Area of shaded region = × 𝜋 × 422 + × 6 × 6
360 2
1 1
∴ Area of shaded region = × 𝜋 × 422 + × 36
4 2
22
Substituting 𝜋 = we get,
7
1 22 1
∴ Area of shaded region = × × 422 + × 36
4 7 2
1
∴ Area of shaded region = × 11 × 6 × 42 + 18
2

∴ Area of shaded region = 11 × 3 × 42 + 18


∴ Area of shaded region = 1386 + 18
∴ Area of shaded region = 1404
Therefore, area of the shaded region is 1404 cm².

T IO S
Page No 13.64:

I
Question 47: In the following figure, ABCD is a trapezium of area 24.5
NO A D
CO N
cm2, If AD || BC, ∠DAB = 90°, AD = 10 cm, BC = 4 cm and ABE is
A
quadrant of a circle, then find the area of the shaded region. [CBSE 2014]

PY
DO U A
T
ED PR
C
©

ANSWER:
1
Area of trapezium = (𝐴𝐷 + 𝐵𝐶 ) × 𝐴𝐵
2
1
⇒ 24.5 = (10 + 4) × 𝐴𝐵
2

⇒ 𝐴𝐵 = 3.5 cm

Area of shaded region = Area of trapezium ABCD − Area of quadrant


ABE
1 22
= 24.5 − × × (3.5)2
4 7

= 24.5 − 9.625
= 14.875 cm2
Hence, the area of shaded region is 14.875 cm2

Page No 13.64:
Question 48: In the given figure, ABCD is a trapezium with AB || DC,
AB = 18 cm DC = 32 cm and the distance between AB and DC is 14 cm.
Circles of equal radii 7 cm with centres A, B, C and D have been drawn.
22
Then find the area of the shaded region.(Use π = )

T IO S
[CBSE 2014]
7

I
NO A D
CO N
A

PY
DO U A
T
ED PR

ANSWER:
Area of shaded region = Area of trapezium ABCD − Area of 4 sectors
C
©

1 ∠𝐴 ∠𝐵 ∠𝐶 ∠𝐷
= (𝐴𝐵 + 𝐷𝐶 ) × 14 − ( 𝜋𝑟 2 + 𝜋𝑟 2 + 𝜋𝑟 2 + 𝜋𝑟 2 )
2 360° 360° 360° 360°
∠𝐴+∠𝐵+∠𝐶+∠𝐷
= 12(𝐴𝐵 + 𝐷𝐶 ) × 14 − ( ) 𝜋𝑟 2
360°
22
= 12(18 + 32) × 14 − (7)2
7

= 350 − 154
= 196 cm2
Hence, the area of shaded region is 196 cm2
Page No 13.64:
Question 49: From a thin metallic piece, in the shape of a
trapezium ABCD, in which AB || CD and ∠BCD = 90°, a quarter
circle BEFC is removed (in the following figure). Given AB = BC = 3.5
cm and DE = 2 cm, calculate the area of the remaining piece of the metal
sheet.

T IO S I
NO A D
CO N
A

PY
DO U A

ANSWER:
T
ED PR

We have given a trapezium. We are asked to find the area of the shaded
region.
C

We can find the area of the remaining part that is area of the shaded
©

region as shown below.


Area of the shaded region = Area of the trapezium – Area of the sector
1 𝜃
∴ Area of the shaded region = (𝐴𝐵 + 𝐶𝐷) × 𝐵𝐶 − 𝜋𝑟 2
2 360
1 90
∴ Area of the shaded region = (3.5 + 𝐶𝐷) × 3.5 − 𝜋(3.5)2
2 360

……. (1)
Now we find the value of CD.
𝐶𝐷 = 𝐶𝐸 + 𝐷𝐸
∴ 𝐶𝐷 = 3.5 + 2 … (Since, CE is radius of the sector, therefore, CE =
3.5)
∴ 𝐶𝐷 = 5.5
22
Substituting the values of CD and 𝜋 = in equation (1),
7
1 90 22
∴ Area of the shaded region = (3.5 + 5.5) × 3.5 − × × (3.5)2
2 360 7
1 90 22
∴ Area of the shaded region = (9) × 3.5 − × × 3.5 × 3.5
2 360 7
31.5 1
∴ Area of the shaded region = − × 22 × 0.5 × 3.5
2 4

T IO S
31.5 1
∴ Area of the shaded region = − × 11 × 0.5 × 3.5
2 2

I
NO A D
∴ Area of the shaded region =
31.5

19.25

CO N
2 2
A
12.25
∴ Area of the shaded region =

PY
2
DO U A

∴ Area of the shaded region = 6.125


T
ED PR

Therefore, area of the remaining part is 6.125 cm².


C
©

Page No 13.65:
Question 50: In the following figure, ABC is an equilateral triangle of
side 8 cm. A, B and C are the centres of circular arcs of radius 4 cm. Find
the area of the shaded region correct upto 2 decimal places. (Take π =
3.142 and √3 = 1.732).
ANSWER:
Area of the shaded region can be calculated as shown below,
Area of the shaded region = Area of equilateral triangle − 3 area of
circular arc
√3 60
∴ Area of the shaded region = ×8×8−3× ×𝜋×4×4
4 360
1
∴ Area of the shaded region = √3 × 2 × 8 − 3 × × 𝜋 × 4 × 4
6

T IO S
1
∴ Area of the shaded region = √3 × 16 − × 𝜋 × 16

I
2

NO A D
∴ Area of the shaded region = √3 × 16 − 𝜋 × 8

CO N
A
Substituting √3 = 1.732 and 𝜋 = 3.142 we get,

PY
DO U A
∴ Area of the shaded region = 1.732 × 16 − 3.142 × 8
T
ED PR

∴ Area of the shaded region = 27.712 − 25.136


∴ Area of the shaded region = 2.576
C

Therefore, area of the shaded region is 2.576 cm².


©

Page No 13.65:
Question 51: Sides of a triangular field are 15 m, 16 m, 17 m. With three
corners of the field a cow, a buffalo and a horse are tied separately with
ropes of length 7 m each to graze in the field. Find the area of the field
which cannot be grazed by three animals.
ANSWER:
T IO S
The area grazed by the cow, buffalo and the horse are in the form of

I
sectors of the circle with radius 7 m.
NO A D
CO N
Let the angle formed in the three sectors be 𝜃1 , 𝜃2 , 𝜃3
A
Now the area of these sectors will be

PY
𝜋 𝜃1 (7)2
DO U A
Area of sector with angle 𝜃1 =
360
T
𝜋 𝜃2 (7)2
ED PR

Area of sector with angle 𝜃2 =


360
𝜋 𝜃3 (7)2
C

Area of sector with angle 𝜃3 =


360
©

Area of the triangle ABC will be


𝑎+𝑏+𝑐 15+16+17 48
𝑠= = = = 24
2 2 2

Area = √𝑠(𝑠 − 𝑎)(𝑠 − 𝑏)(𝑠 − 𝑐)

= √24(24 − 15)(24 − 16)(24 − 17)


= 24√21 m2
Area of the field not grazed by the animals = Area of the triangle ABC −
Area of the three sectors
𝜋 𝜃1 (7)2 𝜋 𝜃2 (7)2 𝜋 𝜃3 (7)2
= 24√21 − [( )+( )+( )]
360 360 360
𝜋 (7)2
= 24√21 − ( ) (𝜃1 + 𝜃2 + 𝜃3 )
360
𝜋 (7)2
= 24√21 − ( ) (180) (Angle sum property)
360

= (24√21 − 77) m2

Page No 13.65:
Question 52: In the given figure, the side of square is 28 cm and radius
of each circle is half of the length of the side of the square where O and
O' are centres of the circles. Find the area of shaded region.

T IO S I
NO A D
CO N
A

PY
DO U A
T
ED PR
C
©

ANSWER:
We have,
28
Side of square = 28 cm and radius of each circle = cm
2
Area of the shaded region

= Area of the square + Area of the two circles − Area of the two
quadrants

28 2 1 28 2
= (28)2 + 2 × 𝜋 × ( ) − 2× ×𝜋×( )
2 4 2

3 28 2
= (28 )2 + ×𝜋×( )
2 2

T IO S
3 22 1 1
= (28)2 (1 + × × × )

I
2 7 2 2

NO A D
CO N
33
= (28)2 (1 + )
A
28
61

PY
= (28)2 ×
DO U A
28
T
= 28 × 61
ED PR

= 1708 cm2
C
©

Therefore, the area of the shaded region is 1708 cm2.

Page No 13.65:
Question 53: In a hospital used water is collected in a cylindrical tank of
diameter 2 m and height 5 m. After recycling, this water is used to irrigate
a park of hospital whose length is 25 m and breadth is 20 m. If tank is
filled completely then what will be the height of standing water used for
irrigating the park.
ANSWER:
Diameter of cylinder (d) = 2 m
Radius of cylinder (r) = 1 m
Height of cylinder (H) = 5 m
Volume of cylindrical tank, Vc = 𝜋𝑟 2 𝐻 = 𝜋 × (1)2 × 5 = 5𝜋 m

Length of the park (l) = 25 m


Breadth of park (b) = 20 m
height of standing water in the park = h
Volume of water in the park = 𝑙𝑏ℎ = 25 × 20 × ℎ

Now water from the tank is used to irrigate the park. So,
Volume of cylindrical tank = Volume of water in the park

T IO S I
⇒ 5𝜋 = 25 × 20 × ℎ
NO A D
CO N
5𝜋
⇒ =ℎ
A
25×20

PY
𝜋
DO U A
⇒ℎ= 𝑚
100
T
ED PR

⇒ ℎ = 0.0314 m
C

VERY SHORT ANSWER TYPE QUESTIONS (VSAQs)


©

Page No 13.67:
Question 1: What is the ratio of the areas of a circle and an equilateral
triangle whose diameter and a side are respectively equal?
ANSWER:
We are given that diameter and side of an equilateral triangle are equal.
Let d and a are the diameter and side of circle and equilateral triangle
respectively.
Therefore, d = a
We know that area of the circle = 𝜋𝑟 2
√3 2
Area of the equilateral triangle = 𝑎
4
Now we will find the ratio of the areas of circle and equilateral triangle.
Area of circle 𝜋𝑟 2
So, = √3
Area of equilateral triangle 𝑎2
4

We know that radius is half of the diameter of the circle.


𝑑 2
Area of circle 𝜋( 2 )
⇒ = √3 2
Area of equilateral triangle 𝑎
4

𝑑2

T IO S
Area of circle 𝜋× 4
= √3 2
Area of equilateral triangle

I
4
𝑎

NO A D
CO N
Now we will substitute 𝑑 = 𝑎 in the above equation,
A
𝑎2
𝜋× 4

PY
Area of circle
=
DO U A
Area of equilateral triangle √3 2
𝑎
T 4
ED PR

Area of circle 𝜋
⇒ =
Area of equilateral triangle √3
C

Therefore, ratio of the areas of circle and equilateral triangle is 𝜋: √3.


©

Page No 13.67:
Question 2: If the circumference of two circles are in the ratio 2: 3, what
is the ratio of their areas?
ANSWER:
We are given ratio of circumferences of two circles. If 𝐶 = 2𝜋𝑟 and
𝐶 ′ = 2𝜋𝑟 ′ are circumferences of two circles such that
𝐶 2
=
𝐶′ 3
2𝜋𝑟 2
⇒ = …… (1)
2𝜋𝑟 ′ 3
Simplifying equation (1) we get,
𝑟 2
=
𝑟′ 3
2
Let 𝐴 = 𝜋𝑟 2 and 𝐴′ = 𝜋𝑟 ′ are the areas of the respective circles and we
are asked to find their ratio.
𝐴 𝜋𝑟 2
= 2
𝐴′ 𝜋𝑟 ′
𝐴 𝑟2
=

T IO S
2
𝐴′ 𝑟′

I
𝐴 𝑟 2
= ( ′) …… (2)
𝐴′
NO A D𝑟

CO N
A
𝑟 2
We know that = substituting this value in equation (2) we get,
𝑟′ 3

PY
DO U A
𝐴 2 2
=( )
T
𝐴′ 3
ED PR

𝐴 4
⇒ =
𝐴′ 9
C

Therefore, ratio of their areas is 4: 9.


©

Page No 13.67:
Question 3: Write the area of the sector of a circle whose radius is r and
length of the arc is l.
ANSWER:
𝜃
We know that area of the sector of the circle of radius r = × 𝜋𝑟 2
360
𝜃
Length of the arc = × 2𝜋𝑟
360

But we have given that length of the arc = l


𝜃
So, 𝑙 = × 2𝜋𝑟 …… (1)
360
𝜃
Area of the sector = × 𝜋𝑟 2
360

Now we will adjust 2 in the following way,


𝜃 2𝜋𝑟 2
Area of the sector = ×
360 2
𝜃 𝑟
Area of the sector = × 2𝜋𝑟 ×
360 2
𝜃
From equation (1) we will substitute × 2𝜋𝑟 = 𝑙
360

T IO S
𝑟
Area of the sector = 𝑙 ×

I
2

NO A D
Area of the sector = 𝑙𝑟
1

CO N
2
A
1
Therefore, area of the sector = 𝑙𝑟.

PY
DO U A
2
T
ED PR

Page No 13.67:
Question 4: What is the length (in terms of π) of the arc that subtends an
C
©

angle of 36° at the centre of a circle of radius 5 cm?


ANSWER:
We have
𝑟 = 5 cm
𝜃 = 36°
We have to find the length of the arc.
𝜃
Length of the arc = × 2𝜋𝑟
360

Substituting the values, we get,


36
Length of the arc = × 2𝜋 × 5 ………. (1)
360
Now we will simplify the equation (1) as below,
1
Length of the arc = × 2𝜋 × 5
10
1
Length of the arc = × 2𝜋
2

Length of the arc = 𝜋


Therefore, length of the arc is 𝜋 cm

T IO S
Page No 13.67:

I
NO A D
Question 5: What is the angle subtended at the centre of a circle of radius

CO N
6 cm by an arc of length 3 π cm?
A

PY
ANSWER:
DO U A
T
We have
ED PR

r = 6 cm
C

length of the arc = 3𝜋 cm


©

We will find the angle subtended at the centre of a circle.


𝜃
Length of the arc = × 2𝜋𝑟
360

Substituting the values, we get,


𝜃
3𝜋 = × 2𝜋 × 6 …… (1)
360

Now we will simplify the equation (1) as below,


𝜃
3𝜋 = × 12𝜋
360
𝜃
3𝜋 = ×𝜋
30
𝜃
3=
30
⇒ 𝜃 = 90°
Therefore, angle subtended at the centre of the circle is 90°.

Page No 13.68:
Question 6: What is the area of a sector of a circle of radius 5 cm formed
by an arc of length 3.5 cm?
ANSWER:

T IO S
We have

I
NO A D
𝑟 = 5 cm

CO N
A
Length of the arc(l) = 3.5 cm

PY
DO U A
Now we will find the area of the sector.
T
1
ED PR

Area of the sector = × 𝑙𝑟


2

Substituting the values, we get,


C

1
©

Area of the sector = × 3.5 × 5 …… (1)


2

Now we will simplify the equation (1) as below,


1
Area of the sector = × 17.5
2

Area of the sector = 8.75


Therefore, area of the sector is 8.75 cm².

Page No 13.68:
Question 7: In a circle of radius 10 cm, an arc subtends an angle of 108°
at the centre. what is the area of the sector in terms of π?
ANSWER:
We have given the radius of the circle and angle subtended at the centre
of the circle.
𝑟 = 10 cm
𝜃 = 108°
Now we will find the area of the sector.
𝜃
Area of the sector = × 𝜋𝑟 2
360

T IO S
Substituting the values, we get,

I
108
Area of the sector = × 𝜋 × 102 …… (1)
NO A D 360

CO N
A
Now we will simplify the equation (1) as below,

PY
3
× 𝜋 × 100
DO U A
Area of the sector =
10
T
Area of the sector = 3 × 𝜋 × 10
ED PR

⇒ Area of the sector = 30𝜋


C

Therefore, area of the sector is 30𝜋 cm².


©

Page No 13.68:
Question 8: If a square is inscribed in a circle, what is the ratio of the
areas of the circle and the square?
ANSWER:
We have the following situation
Let BD be the diameter and diagonal of the circle and the square
respectively.

T IO S
We know that area of the circle = 𝜋𝑟 2

I
NO A D
Area of the square = side2

CO N
A
As we know that diagonal of the square is the diameter of the square.

PY
DO U A
Diagonal = 2r
T
diagonal
ED PR

Side of the square = …… (1)


√2

Substituting Diagonal = 2r in equation (1) we get,


C

2r
©

Side of the square =


√2

Now we will find the ratio of the areas of circle and square.
Area of circle 𝜋𝑟 2
= 2r 2
Area of square ( )
√2

Now we will simplify the above equation as below,


Area of circle 𝜋𝑟 2
= 4𝑟2
Area of square
2

Area of circle 2
= 𝜋𝑟 2 ×
Area of square 4𝑟 2
Area of circle 𝜋
Hence, =
Area of square 2
Therefore, ratio of areas of circle and square is 𝜋: 2.

Page No 13.68:
Question 9: Write the formula for the area of a sector of angle θ (in
degrees) of a circle of radius r.
ANSWER:
Let r be the radius of the circle and angle θ subtended at the centre of the

T IO S
circle.

I
𝜃
NO A D
Area of the sector of the circle = × 𝜋𝑟 2

CO N
360
A
𝜃
Therefore, area of the sector is × 𝜋𝑟 2 .

PY
360
DO U A
T
ED PR

Page No 13.68:
Question 10: Write the formula for the area of a segment in a circle of
C

radius r given that the sector angle is θ (in degrees).


©

ANSWER:

In this figure, centre of the circle is O, radius OA = r and ∠𝐴𝑂𝐵 = 𝜃


We are going to find the area of the segment AXB.
Area of the segment AXB = Area of the sector OAXB – Area of ∆AOB
…… (1)
𝜃
We know that area of the sector OAXB = × 𝜋𝑟 2
360
𝜃 𝜃
We also know that area of ∆AOB = 𝑟 2 sin cos
2 2

Substituting these values in equation (1) we get,


𝜃 𝜃 𝜃
Area of the segment AXB = × 𝜋𝑟 2 − 𝑟 2 sin cos

T IO S
360 2 2

I
𝜃 𝜃 𝜃
Area of the segment AXB = ( × 𝜋 − sin cos ) 𝑟 2
NO A D 360 2 2

CO N
A
𝜋𝜃 𝜃 𝜃
Area of the segment AXB = ( − sin cos ) 𝑟 2
360 2 2

PY
DO U A
𝜋𝜃 𝜃 𝜃
Therefore, area of the segment is ( − sin cos ) 𝑟 2 .
T 360 2 2
ED PR

Page No 13.68:
C
©

Question 11: If the adjoining figure is a sector of a circle of radius 10.5


22
cm, what is the perimeter of the sector? (Take 𝜋 = )
7

ANSWER:
Given figure is a quadrant of a circle. We have given radius of sector
that is 10.5 cm. Arc AB subtended an angle of 60° at the centre of the
circle.
𝜃
Perimeter of the sector = × 2𝜋𝑟 + 2𝑟
360

Substituting the values, we get,


60
Perimeter of the sector = × 2𝜋 × 10.5 + 2 × 10.5 …… (1)
360

T IO S
Now we will simplify equation (1) as shown below,

I
1
Perimeter of the sector = × 2𝜋 × 10.5 + 2 × 10.5
NO A D 6

CO N
1
A
Perimeter of the sector = × 21𝜋 + 21
6

PY
DO U A
22
Now we will substitute 𝜋 = .
7
T
ED PR

1 22
Perimeter of the sector = × 21 × + 21
6 7
1
C

Perimeter of the sector = × 3 × 22 + 21


6
©

1
Perimeter of the sector = × 22 + 21
2

Perimeter of the sector = 11 + 21


Perimeter of the sector = 32
Therefore, perimeter of the given sector is 32 cm.

Page No 13.68:
Question 12: If the diameter of a semi-circular protractor is 14 cm, then
find its perimeter.
ANSWER:
Let AB be the diameter of the semi-circular protractor.
So, AB = 14 cm
1
We know that perimeter of the semicircle = (2𝜋𝑟) + 2𝑟 ……(1)
2

We have given the diameter of the protractor.


14
Therefore, radius of the protractor =
2

So, radius of the protractor = 7

T IO S
Substituting the value of r in equation (1) we get,

I
NO A D 1
Perimeter of the semi-circular protractor = (2𝜋 × 7) + 2 × 7

CO N
2
A
Perimeter of the semi-circular protractor = (𝜋 × 7) + 14

PY
DO U A
22
Substituting 𝜋 = we get,
T
7
ED PR

22
Perimeter of the semi-circular protractor = ( × 7) + 14
7
C

Perimeter of the semi-circular protractor = 22 + 14


©

Hence perimeter of the semi-circular protractor = 36


Therefore, perimeter of the semi-circular protractor is 36 cm.

Page No 13.68:
Question 13: An arc subtends an angle of 90° at the centre of the circle
of the radius 14 cm. Write the area of minor sector thus formed in terms
of π.
ANSWER:
We have given an angle subtended by an arc at the centre of the circle
and radius of the circle.
𝑟 = 14 cm
𝜃 = 90°
Now we will find the area of the minor sector.
𝜃
Area of the minor sector = × 𝜋𝑟 2
360

Substituting the values, we get,


90
Area of the minor sector = × 𝜋 × 142 …… (1)
360

T IO S
Now we will simplify the equation (1) as below,

I
NO A D 1
Area of the minor sector = × 𝜋 × 142

CO N
4
A
1
Area of the minor sector = × 𝜋 × 14 × 14

PY
DO U A
4

Area of the minor sector = π × 7 × 7


T
ED PR

Area of the minor sector = 49π


Therefore, area of the minor sector is 49𝜋 cm².
C
©

Page No 13.68:
Question 14: Find the area of the largest triangle that can be inscribed in
a semi-circle of radius r units. [CBSE 2015]
ANSWER:
Radius of the semi-circle r units
When a largest triangle inscribed in a semicircle, then base = r + r =
2r units
Thus, Area of triangle is given by
1
× base × height
2
1
= × (2r) × r
2

= r 2 square units

Page No 13.68:
Question 15: Find the area of sector of circle of radius 21 cm and central
angle 1200.

T IO S
ANSWER:

I
𝜃
Area of a sector of a circle = × 𝜋𝑟 2
NO A D 360

CO N
120
= × 𝜋(21 )2
A
360

PY
= 462 cm2
DO U A
T
Page No 13.68:
ED PR

Question 16: What is the area of a square inscribed in a circle of


diameter p cm?
C
©

ANSWER:
The diameter of the circle = Diagonal of the square = p
We know that diagonal of a square = √2𝑎
⇒ √2𝑎 = 𝑝
𝑝
⇒𝑎=
√2
𝑝 2 𝑝2
Thus, the area of the square = 𝑎2 = ( ) = cm2
√2 2

Page No 13.68:
Question 17: Is it true to say that area of segment of a circle is less than
the area of its corresponding sector? Why?
ANSWER:
A circle has two segments, a major segment and a minor segment.
The area of the minor segment is less than the area of the corresponding
sector.
But same is not true for the major segment.

Page No 13.68:

T IO S
Question 18: If the numerical value of the area of a circle is equal to the
numerical value of its circumference, find its radius.

I
NO A D
ANSWER:

CO N
A
Given that area of the circle = circumference

PY
𝜋𝑟 2 = 2𝜋𝑟
DO U A
T
⇒ 𝜋𝑟 2 − 2𝜋𝑟 = 0
ED PR

⇒ 𝜋𝑟(𝑟 − 2) = 0
C

⇒ 𝑟 = 0 𝑜𝑟 𝑟 = 2
©

Since radius cannot be equal to 0 so, r = 2

Page No 13.68:
Question 19: How many revolutions a circular wheel of radius r metres
makes in covering a distance of s metres?
ANSWER:
In one revolution, the wheel covers a distance of 2πr
𝑠
So, to cover a distance of s meters, the wheel has to make revolutions.
2𝜋𝑟
Page No 13.68:
Question 20: Find the ratio of the area of the circle circumscribing a
square to the area of the circle inscribed in the square.
ANSWER:

T IO S I
NO A D
CO N
A
Let the side of the square inscribed in a square be a units.

PY
DO U A
Diameter of the circle outside the square = Diagonal of the square = √2𝑎
T
√2𝑎 𝑎
Radius = =
ED PR

2 √2
𝑎 2
So, the area of the circle circumscribing the square = 𝜋 ( ) .....(i)
√2
C

𝑎
Now, the radius of the circle inscribed in a square =
©

2
𝑎 2
Hence, area of the circle inscribed in a square = 𝜋 ( ) .....(ii)
2
From (i) and (ii)
𝑎 2
Area of circle circumscribing a square 𝜋( )
√2
= 𝑎 2
Area of circle inscribed in a square 𝜋( 2 )

1
2
= 1
4
2
=
1
Hence, the required ratio is 2 : 1.
MULTIPLE CHOICE QUESTION
Page No 13.69:
Question 1: If the circumference and the area of a circle are numerically
equal, then diameter of the circle is

𝜋
(a)
2

(b) 2π

T IO S I
(c) 2
NO A D
CO N
A
(d) 4

PY
DO U A
ANSWER:
T
We have given that circumference and area of a circle are numerically
ED PR

equal.
C

Let it be x.
©

Let r be the radius of the circle, therefore, circumference of the circle is


2𝜋𝑟 and area of the circle will be 𝜋𝑟 2 .
Therefore, from the given condition we have,
2𝜋𝑟 = 𝑥 ……… (1)
𝜋𝑟 2 = 𝑥 ……… (2)
𝑥
Therefore, from equation (1) get 𝑟 = . Now we will substitute this
2𝜋
𝑥 2
value in equation (2) we get, 𝜋 ( ) = 𝑥
2𝜋

Simplifying further we get,


𝑥2
𝜋× =𝑥
4𝜋2
Cancelling x, we get,
𝑥
𝜋× =1
4𝜋2

Now we will cancel 𝜋


𝑥
= 1 ……… (3)
4𝜋2

Now we will multiply both sides of the equation (3) by 4 𝜋 we get, 𝑥 =


4𝜋

T IO S
We can rewrite this equation as given below, 𝑥 = 2 × 𝜋 × 2

I
Comparing equation (4) with equation (1) we get r = 2.
NO A D
CO N
Therefore, radius of the circle is 2. We know that diameter of the circle
A
is twice the radius of the circle.

PY
DO U A
∴ diameter = 2 × radius
T
ED PR

∴ diameter = 2 × 2
∴ diameter = 4
C
©

Therefore, diameter of the circle is 4.


Hence, option (d) is correct.

Page No 13.69:
Question 2: If the difference between the circumference and radius of a
22
circle is 37 cm, then using π = , the circumference (in cm) of the circle
7
is

(a) 154

(b) 44
(c) 14

(d) 7
ANSWER:
We know that circumference; C of the circle with radius r is equal to 2πr.
We have given difference between circumference and radius of the circle
that is 37 cm.

T IO S
∴ 𝐶 − 𝑟 = 2πr − r

I
∴ (2π − 1)𝑟 = 37
NO A D
CO N
22
Substituting π =
A
we get,
7

PY
22
DO U A
∴ (2 × − 1) 𝑟 = 37
7
T
ED PR

44−7
∴( ) 𝑟 = 37
7
37
∴ ( ) 𝑟 = 37
C

7
©

7
Dividing both sides of the equation by , we get, ∴ 𝑟 = 7
37

Therefore, circumference of the circle will be


22
2𝜋𝑟 = 2 × ×7
7

= 44 cm2
Hence, the correct choice is (b).

Page No 13.69:
Question 3: A wire can be bent in the form of a circle of radius 56 cm.
If it is bent in the form of a square, then its area will be
(a) 3520 cm2

(b) 6400 cm2

(c) 7744 cm2

(d) 8800 cm2


ANSWER:

T IO S
We have given that a wire is bent in the form of circle of radius 56 cm. If

I
we bent the same wire in the form of square of side a cm, the perimeter of
NO A D
the wire will not change.

CO N
A
∴ perimeter of the circle = perimeter of the square

PY
DO U A
∴ 2𝜋𝑟 = 4𝑎
T
ED PR

We know that r = 56 cm.


Now we will substitute the value of r in the equation,
C

2 × 𝜋 × 56 = 4𝑎 ……… (1)
©

22
∴2× × 56 = 4𝑎
7

∴ 2 × 22 × 8 = 4𝑎
Dividing both sides of the equation by 4 we get,
2×22×8
𝑎=
4

∴ 𝑎 = 2 × 22 × 2
∴ 𝑎 = 88
Now we obtained side of the square. Now we can calculate the area of
the square as given below.
Area of the square = 𝑎2
∴ Area of the square = 882
∴ Area of the square = 7744
Hence, the area of the square is 7744 cm².
Therefore, the correct answer is (c).

Page No 13.69:

T IO S
Question 4: If a wire is bent into the shape of a square, then the area of
the square is 81 cm2. When wire is bent into a semi-circular shape, then

I
NO A D
the area of the semi-circle will be

CO N
A
(a) 22 cm2

PY
DO U A
(b) 44 cm2
T
ED PR

(c) 77 cm2
C

(d) 154 cm2


©

ANSWER:
We have given that a wire is bent in the form of square of side a cm such
that the area of the square is 81 cm². If we bent the same wire in the form
of a semicircle with radius r cm, the perimeter of the wire will not change.
∴ perimeter of the square = perimeter of semi-circle
1
4𝑎 = (2𝜋𝑟) + 2𝑟 ……… (1)
2

We know that area of the square = 81 cm².


∴ 𝑎2 = 81
∴ 𝑎=9
Now we will substitute the value of 𝑎 in the equation (1),
1
4 × 9 = (2𝜋𝑟) + 2𝑟
2
1
∴ 36 = (2𝜋𝑟) + 2𝑟
2

∴ 36 = (𝜋𝑟) + 2𝑟
∴ 36 = 𝑟(𝜋 + 2)
22
Now we will substitute 𝜋 = .
7
22

T IO S
∴ 36 = 𝑟 ( + 2)
7

I
22+14
NO A D
∴ 36 = 𝑟 ( )

CO N
7
A
36
∴ 36 = 𝑟 ( )

PY
7
DO U A

Multiplying both sides of the equation by 7 we get, 36 × 7 = 𝑟 × 36


T
ED PR

Now we will divide both sides of the equation by 36 we get, r = 7


Therefore, radius of the semi-circle is 7cm.
C
©

Now we will find the area of the semicircle.


1
Area of the semicircle = × 𝜋𝑟 2
2
1
= × 𝜋 × 72
2
1 22
= × × 72
2 7

= 11 × 7
= 77
Therefore, area of the semicircle is 77 cm².
Hence the correct answer is option (c).
Page No 13.69:
Question 5: A circular park has a path of uniform width around it. The
difference between the outer and inner circumferences of the circular path
is 132 m. Its width is

(a) 20 m

(b) 21 m

T IO S I
(c) 22 m
NO A D
CO N
A
(d) 24 m

PY
ANSWER:
DO U A
T
ED PR
C
©

Let OA = r be the radius of the inner circle and OB = r′ be the radius of


the outer circle.
Therefore, circumference of the inner circle = 2𝜋𝑟 and circumference of
the outer circle = 2𝜋𝑟 ′
Here we have to find the width of the circular park that is we have to
find 𝑟 ′ − 𝑟.
We have given the difference between the circumferences of outer circle
and inner circle.
∴ 2𝜋𝑟 ′ − 2𝜋𝑟 = 132
∴ 2𝜋(𝑟 ′ − 𝑟) = 132
22
Substituting π = we get,
7
22
2× × (𝑟 ′ − 𝑟) = 132
7
7
Now we will multiply both sides of the equation by .
44
7
(𝑟 ′ − 𝑟) = 132 ×

T IO S
44

I
∴ (𝑟 ′ − 𝑟 ) = 3 × 7
NO A D
CO N
∴ (𝑟 ′ − 𝑟) = 21
A

PY
Therefore, the width is 21 m.
DO U A
T
Hence the correct answer is option (b).
ED PR

Page No 13.69:
C
©

Question 6: The radius of a wheel is 0.25 m. The number of revolutions


it will make to travel a distance of 11 km will be

(a) 2800

(b) 4000

(c) 5500

(d) 7000
ANSWER:
We have given the radius of the wheel that is 0.25 cm.
We know that distance covered by the wheel in one revolution =
Distance moved
.
Number of revolutions

Distance covered by the wheel in one revolution is equal to the


circumference of the wheel.
Distance moved
2𝜋𝑟 = ……… (1)
Number of revolutions

Distance moved is given as 11 km so we will first convert it to m.

T IO S
11 km = 11000 m

I
Now we will substitute the values in equation (1),
NO A D
CO N
11000
2 × 𝜋 × 0.25 =
A
Number of revolutions

PY
22
Now we will substitute π =
DO U A
.
7
T
22 11000
ED PR

2× × 0.25 =
7 Number of revolutions

Simplifying equation (1) we get,


C

11000×7
©

Number of revolution =
2×22×0.25
11000×7
∴ Number of revolution =
22×0.5
1000×7
∴ Number of revolution =
2×0.5
7000
∴ Number of revolution =
1

∴ Number of revolution = 7000


Therefore, it will make 7000 revolutions to travel a distance of 11 km.
Hence, the correct answer is option (d).
Page No 13.69:
Question 7: The ratio of the outer and inner perimeters of a circular path
is 23 : 22. If the path is 5 metres wide, the diameter of the inner circle is

(a) 55 m

(b) 110 m

(c) 220 m

T IO S I
NO A D
(d) 230 m

CO N
A
ANSWER:

PY
DO U A
T
ED PR
C
©

Let OA = r be the radius of the inner circle and OB = r′ be the radius of


the outer circle.
Therefore, circumference of the inner circle = 2𝜋𝑟 and circumference of
the outer circle = 2𝜋𝑟 ′
Here we have to find the diameter of the inner circle.
We have given the ratio of outer and inner perimeters of a circular path.
2𝜋𝑟 ′ 23
∴ =
2𝜋𝑟 22

Simplifying the above equation, we get,


𝑟′ 23
=
𝑟 22

∴ 22𝑟 ′ = 23𝑟
∴ 22𝑟 ′ − 23𝑟 = 0 …….. (1)
We have also given that the path is 5 meters wide, that is we have given
𝑟′ − 𝑟 = 5 …….. (2)
We are asked to find the diameter of the inner circle hence, we will

T IO S
eliminate r′ using equations (1) and (2) for that we will multiply equation
(2) by 22.

I
NO A D
22𝑟 ′ − 23𝑟 = 110 …….. (3)

CO N
A
Subtracting equation (1) from equation (3) we get, r = 110

PY
DO U A
Therefore, radius of the inner circle is 110 meters.
T
ED PR

Therefore, diameter of the inner circle = 2 × 110 = 220 meters


Therefore, diameter of the inner circle is 220 meters.
C

Hence, the correct answer is option (c).


©

Page No 13.69:
Question 8: The circumference of a circle is 100 m. The side of a square
inscribed in the circle is
(a) 50√2
50
(b)
𝜋

50√2
(c)
𝜋

100√2
(d)
𝜋
ANSWER:

We have given the circumference of the circle that is 100 cm. If d is the

T IO S
diameter of the circle, then its circumference will be 𝜋𝑑.

I
NO A D
∴ 𝜋𝑑 = 100

CO N
A
100
∴ 𝑑=
𝜋

PY
DO U A
We obtained diameter of the circle. Look at the figure, diameter of the
T
circle is also the diagonal of the square ABCD.
ED PR

We know that if we have diagonal of the circle we can calculate the side
C

of the square, using the formula given below,


©

Side = √2 × diagonal
Substituting the value of diagonal we get,
100
Side = √2 ×
𝜋
100√2
Therefore, side of the inscribed square is cm.
𝜋

Hence, the correct answer is option (d).

Page No 13.69:
Question 9: The area of the in circle of an equilateral triangle of side 42
cm is
(a) 22√3 cm2

(b) 231 cm2

(c) 462 cm2

(d) 924 cm2

T IO S
ANSWER:

I
Let ABC be the equilateral triangle such that AB = BC = CA = 42 cm.
NO A D
Also, let O be the centre and r be the radius of its in circle.

CO N
A

PY
DO U A
T
ED PR
C
©

AB, BC and CA are tangents to the circle at M, N and P.


∴ OM = ON = OP = r
Area of ΔABC = Area (ΔOAB) + Area (ΔOBC) + Area (ΔOCA)
√3 1 1 1
⇒ (42)2 = × 𝑟 × 𝐴𝐵 × + × 𝑟 × 𝐵𝐶 + × 𝑟 × 𝐶𝐴
4 2 2 2

√3 1
⇒ × 42 × 42 = 𝑟(𝐴𝐵 + 𝐵𝐶 + 𝐶𝐴)
4 2
1
⇒ 441√3 = × 𝑟 × (42 + 42 + 42)
2
1
⇒ 441√3 = × 𝑟 × (126)
2

⇒ 441√3 = 63𝑟
441√3
⇒𝑟=
63

⇒ 𝑟 = 7√3 cm
22 2 22
Area of the circle = 𝜋𝑟 2 = (7√3) = × 147 = 462 cm²
7 7

T IO S
Hence, the correct answer is option (c).

I
NO A D
CO N
A
Page No 13.69:

PY
Question 10: The area of in circle of an equilateral triangle is 154 cm2.
DO U A
The perimeter of the triangle is
T
ED PR

(a) 71.5 cm
C

(b) 71.7 cm
©

(c) 72.3 cm

(d) 72.7 cm
ANSWER:
Area of in circle of equilateral triangle is 154 cm2
T IO S I
We have to find the perimeter of the triangle. So we will use area to get,
NO A D
CO N
Area of in circle = 154
A
𝜋𝑟 2 = 154

PY
DO U A

154
T
𝑟=√ cm
ED PR

As triangle is equilateral so,


C

∠𝑂𝐶𝑀 = 30°
©

So,
𝑟
tan 30° =
𝑀𝐶

154(3)
𝑀𝐶 = √ cm
𝜋

So,
AC = 2(MC)
154(3)
= 2 (√ ) cm
𝜋

Therefore, perimeter of the triangle is,


= 3(AC)
462
= 6 (√ )
3.14

= 72.7 cm
Therefore, the answer is (d).

Page No 13.69:

T IO S I
Question 11: The area of the largest triangle that can be inscribed in a
NO A D
semi-circle of radius r, is

CO N
A
(a) r2

PY
DO U A
T
(b) 2r2
ED PR

(c) r3
C
©

(d) 2r3
ANSWER:
The triangle with the largest area will be symmetrical as shown in the
figure.

Let the radius of the circle be r.


Hence,
1
𝑎𝑟(∆𝐴𝐵𝐶 ) = (𝑟)(2𝑟)
2

= 𝑟 2 sq. unit
Therefore, the answer is (a).

Page No 13.69:

T IO S
Question 12: The perimeter of a triangle is 30 cm and the circumference

I
of its in circle is 88 cm. The area of the triangle is
NO A D
CO N
A
(a) 70 cm2

PY
DO U A
(b) 140 cm2
T
ED PR

(c) 210 cm2


C

(d) 420 cm2


©

ANSWER:
We have to find the area of the given triangle.
Perimeter of triangle is 30 cm.
Let the radius of the circle be r.
We have,
Circumference of in circle = 88
2𝜋𝑟 = 88
𝑟 = 14

T IO S
Therefore,

I
𝑎𝑟(∆𝐴𝐵𝐶 ) = 𝑎𝑟(∆𝑂𝐴𝐵) + 𝑎𝑟(∆𝑂𝐵𝐶 ) + 𝑎𝑟(∆𝑂𝐴𝐶 )
NO A D
CO N
1
= (𝑟)(𝐴𝐵 + 𝐵𝐶 + 𝐶𝐴)
A
2

PY
1
= (14)(30) cm²
DO U A
2
T
= 210 cm²
ED PR

Therefore, the answer is (c).


C
©

Page No 13.69:
Question 13: The area of a circle is 220 cm2. The area of ta square
inscribed in it is
(a) 49 cm2
(b) 70 cm2
(c) 140 cm2
(d) 150 cm2

ANSWER:
Let BD be the diameter and diagonal of the circle and the square
respectively.

T IO S
We know that area of the circle = 𝜋𝑟 2

I
NO A D
Area of the circle = 𝜋𝑟 2

CO N
A
22
∴ 220 = × 𝑟2
7

PY
DO U A
Multiplying both sides of the equation by 7 we get,
T
ED PR

220 × 7 = 22 × 𝑟 2
Dividing both sides of the equation by 22 we get,
C

∴ 𝑟 2 = 70
©

∴ 𝑟 = √70
As we know that diagonal of the square is the diameter of the square.
∴ Diagonal = 2𝑟
∴ Diagonal = 2√70
diagonal
∴ side of the square = ……… (1)
√2

Substituting Diagonal = 2√70 in equation (1) we get,


2√70
Side of the square =
√2
70
∴ Side of the square = 2√
2

∴ Side of the square = 2√35


∴ Area of the square = side²
2
= (2√35)
= 4 × 35

T IO S
= 140

I
Therefore, area of the square is 140 cm².
NO A D
CO N
Hence, the correct answer is option (c).
A

PY
DO U A

Page No 13.70:
T
ED PR

Question 14: If the circumference of a circle increases from 4π to 8π, then


its area is
C

(a) halved
©

(b) doubled

(c) tripled

(d) quadrupled
ANSWER:
Let the circumference 𝐶 = 4𝜋
∴ 2𝜋𝑟 = 4𝜋
∴𝑟=2
Therefore, area of the circle when radius of the circle is 2 can be calculated
as below,
22
𝜋𝑟 2 = ×4 …….. (1)
7

Now when circumference is 𝐶 = 8𝜋, then the radius of the circle is


calculated as below,
∴ 2𝜋𝑟 = 8𝜋
∴ 𝑟=4

T IO S
Therefore, area of the circle when radius of the circle is 2 can be calculated

I
NO A D
as below,

CO N
A
22
𝜋𝑟 2 = × 16
7

PY
DO U A
22
∴ 𝜋𝑟 2 = 4 ( × 4) …….. (2)
T 7
ED PR

Therefore, from equation (1) and (2) we can say that its area is quadrupled.
Hence, the correct answer is option (d).
C
©

Page No 13.70:
Question 15: If the radius of a circle is diminished by 10%, then its area
is diminished by

(a) 10%

(b) 19%

(c) 20%

(d) 36%
ANSWER:
Let x be the initial radius of the circle.
Therefore, its area is 𝜋𝑟 2 ……..(1)
It is given that the radius is diminished by 10%, therefore, its new radius
is calculated as shown below,
new radius = 𝑥 − 0.10𝑥
∴ new radius = 0.90𝑥
∴ new area = (0.90𝑥 )2

T IO S
∴ new area = 0.81𝑥 2

I
NO A D
Now we will find the percentage decreased in the area.

CO N
A
∴ change = 0.81𝑥 2 − 𝑥 2

PY
DO U A
∴ change = 0.19𝑥 2
T
ED PR

Therefore, its area is diminished by 19%.


Hence, the correct answer is option (b).
C
©

Page No 13.70:
Question 16: If the area of a square is same as the area of a circle, then
the ratio of their perimeters, in terms of π, is

(a) π : √3

(b) 2 : √𝜋

(c) 3 : ππ

(d) π : √2
ANSWER:
We have given that area of a circle of radius r is equal to the area of a
square of side a.
∴ 𝜋𝑟 2 = 𝑎2
∴ 𝑎 = √𝜋𝑟
We have to find the ratio of the perimeters of circle and square.
Perimeter of circle 2𝜋𝑟
∴ = ……… (1)
Perimeter of square 4𝑎

T IO S
Now we will substitute 𝑎 = √𝜋𝑟 in equation (1).

I
Perimeter of circle 2𝜋𝑟
NO A D
Perimeter of square
=
4√𝜋𝑟

CO N
A
Perimeter of circle 𝜋
∴ =

PY
Perimeter of square 2√𝜋
DO U A

Perimeter of circle √𝜋
T
∴ =
ED PR

Perimeter of square 2

Therefore, ratio of their perimeters is √𝜋: 2.


C

Hence, the correct answer is (b).


©

Page No 13.70:
Question 17: The area of the largest triangle that can be inscribed in a
semi-circle of radius r is
(a) 2r

(b) r2

(c) r

(d) √𝑟
ANSWER:
The triangle with the largest area will be symmetrical as shown in the
figure.

T IO S I
NO A D
Let the radius of the circle be r.

CO N
A
Hence,

PY
DO U A
1
𝑎𝑟(∆𝐴𝐵𝐶 ) = (𝑟)(2𝑟)
2
T
ED PR

= 𝑟 2 sq. unit
Therefore, the answer is (b).
C
©

Page No 13.70:
Question 18: The ratio of the areas of a circle and an equilateral triangle
whose diameter and a side are respectively equal, is
(a) 𝜋: √2

(b) 𝜋: √3

(c) √3: 𝜋

(d) √2: 𝜋
ANSWER:
We are given that diameter and side of an equilateral triangle are equal.
Let d and a are the diameter and side of circle and equilateral triangle
respectively.
∴𝑑=𝑎
We know that area of the circle = 𝜋𝑟 2
√3 2
Area of the equilateral triangle = 𝑎
4

T IO S
Now we will find the ratio of the areas of circle and equilateral triangle.

I
Area of circle 𝜋𝑟 2

NO A D
Area of equilateral triangle
= √3

CO N
4
𝑎2
A
We know that radius is half of the diameter of the circle.

PY
DO U A
𝑑 2
Area of circle 𝜋( )
T 2
∴ =
ED PR

Area of equilateral triangle √3 2


𝑎
4

𝑑2
𝜋× 4
C

Area of circle
∴ = √3 2
Area of equilateral triangle
©

4
𝑎

Now we will substitute 𝑑 = 𝑎 in the above equation,


𝑎2
Area of circle 𝜋× 4
= √3 2
Area of equilateral triangle 𝑎
4

Area of circle 𝜋
∴ =
Area of equilateral triangle √3

Therefore, ratio of the areas of circle and equilateral triangle is 𝜋: √3.


Hence, the correct answer is option (b).

Page No 13.70:
Question 19: If the sum of the areas of two circles with radii r1 and r2 is
equal to the area of a circle of radius r, then 𝑟12 + 𝑟22
(a) >r2

(b) =r2

(c) <r2

(d) None of these

T IO S
ANSWER:

I
NO A D
We have given area of the circle of radius r1 plus area of the circle of

CO N
A
radius r2 is equal to the area of the circle of radius r.

PY
Therefore, we have,
DO U A
T
𝜋𝑟12 + 𝜋𝑟22 = 𝜋𝑟 2
ED PR

Cancelling 𝜋, we get
C

𝑟12 + 𝑟22 = 𝑟 2
©

Therefore, 𝑟12 + 𝑟22 = 𝑟 2 .


Hence, the correct answer is option (b).

Page No 13.70:
Question 20: If the perimeter of a semi-circular protractor is 36 cm, then
its diameter is

(a) 10 cm

(b) 12 cm

(c) 14 cm
(d) 16 cm
ANSWER:
We know that perimeter of a semi-circle of radius
1
r = (2𝜋𝑟) + 2𝑟 ……… (1)
2

We have given the perimeter of the semi-circle and we are asked to find
the diameter of the semi-circle.

T IO S
Therefore, substituting the perimeter of the semi-circle in equation (1)

I
NO A D
we get,

CO N
A
1
36 = (2𝜋𝑟) + 2𝑟
2

PY
DO U A
Multiplying both sides of the equation by 2 we get,
T
ED PR

72 = 2𝜋𝑟 + 4𝑟
22
Substituting 𝜋 = we get,
7
C

22
©

72 = 2 × 𝑟 + 4𝑟
7
44
∴ 72 = 𝑟 + 4𝑟
7

Now we will multiply both sides of the equation by 7.


504 = 44𝑟 + 28𝑟
Adding like terms we get,
∴ 504 = 72𝑟
Dividing both sides of the equation 72 we get, 𝑟 = 7
Therefore, radius of the semi-circle is 7cm.
Now we will find the diameter.
Diameter = 2 × 𝑟
∴ Diameter = 2 × 7
∴ Diameter = 14
Hence, diameter of the semi-circle is 14 cm.
Therefore, the correct answer is (c).

T IO S
Page No 13.70:

I
Question 21: The perimeter of the sector OAB shown in the following
NO A D
figure, is

CO N
A
64

PY
(a) cm
DO U A
3
T
ED PR

(b) 26 cm

64
C

(c) cm
5
©

(d) 19 cm

ANSWER:
We know that perimeter of a sector of radius
𝜃
r = 2𝑟 + × 2𝜋𝑟 ………(1)
360

We have given sector angle and radius of the sector and we are asked to
find perimeter of the sector OAB.
Therefore, substituting the corresponding values of the sector angle and
radius in equation (1) we get,
60
Perimeter = 2 × 7 + × 2𝜋 × 7 ……… (2)
360

T IO S
We will simplify equation (2) as shown below,

I
1
Perimeter = 2 × 7 + × 2𝜋 × 7
NO A D 6

CO N
22
A
Substituting 𝜋 = we get,
7

PY
DO U A
1 22
Perimeter = 2 × 7 + × 2 × ×7
6 7
T
ED PR

1
∴ Perimeter = 2 × 7 + × 2 × 22
6
1
C

∴ Perimeter = 2 × 7 + × 22
3
©

22
∴ Perimeter = 2 × 7 +
3
22
∴ Perimeter = 14 +
3

Now we will make the denominator same.


42 22
∴ Perimeter = +
3 3
42+22
∴ Perimeter =
3
64
∴ Perimeter =
3
64
Therefore, perimeter of the sector is cm.
3

Hence, the correct answer is option (a).


Page No 13.70:
Question 22: If the perimeter of a sector of a circle of radius 6.5 cm is
29 cm, then its area is

(a) 58 cm2

(b) 52 cm2

(c) 25 cm2

T IO S I
(d) 56 cm2
NO A D
CO N
A
ANSWER:

PY
We know that perimeter of a sector of radius
DO U A

𝜃
T
r = 2𝑟 + × 2𝜋𝑟 ………(1)
ED PR

360

We have given perimeter of the sector and radius of the sector and we
C

are asked to find the area of the sector. For that we have to find the
©

sector angle.
Therefore, substituting the corresponding values of perimeter and radius
in equation (1) we get,
𝜃
29 = 2 × 6.5 + × 2𝜋 × 6.5 ……… (2)
360

We will simplify equation (2) as shown below,


𝜃
29 = 2 × 6.5 (1 + × 𝜋)
360

Dividing both sides of the equation by 2 × 6.5, we get,


29 𝜃
= (1 + × 𝜋)
2×6.5 360

Subtracting 1 from both sides of the equation we get,


29 𝜃
=1+ ×𝜋 ……… (3)
2×6.5 360
𝜃
We know that area of the sector = × 𝜋𝑟 2
360

From equation (3), we get


29
Area of the sector = ( − 1) 𝑟 2
2×6.5

Substituting r = 6.5 we get,


29

T IO S
Area of the sector = ( − 1) 6.52
2×6.5

I
29×6.52
NO A D
∴ Area of the sector = ( − 6.52 )

CO N
2×6.5
A
29×6.5
∴ Area of the sector = ( − 6.52 )

PY
2
DO U A

188.5
T
∴ Area of the sector = ( − 42.25)
ED PR

∴ Area of the sector = (94.25 − 42.25)


C

∴ Area of the sector = 52


©

Therefore, area of the sector is 52 cm².


Hence, the correct answer is option (b).

Page No 13.70:
Question 23: If the area of a sector of a circle bounded by an arc of length
5π cm is equal to 20π cm2, then its radius is
(a) 12 cm

(b) 16 cm

(c) 8 cm
(d) 10 cm
ANSWER:
We have given length of the arc and area of the sector bounded by that
arc and we are asked to find the radius of the circle.
𝜃
We know that area of the sector = × 𝜋𝑟 2 .
360
𝜃
Length of the arc = × 2𝜋𝑟 2

T IO S
360

I
Now we will substitute the values.
NO A D
CO N
𝜃
Area of the sector = × 𝜋𝑟 2
A
360

PY
𝜃
DO U A
20𝜋 = × 𝜋𝑟 2 …….. (1)
360
T
ED PR

𝜃
Length of the arc = × 2𝜋𝑟
360
𝜃
5𝜋 = × 2𝜋𝑟 …….. (2)
C

360
©

Now we will divide equation (1) by equation (2),


𝜃
20𝜋 360
×𝜋𝑟 2
= 𝜃
5𝜋 ×2𝜋𝑟
360

Now we will cancel the like terms.


20 𝑟2
=
5 2𝑟
𝑟
∴4=
2

∴𝑟=8
Therefore, radius of the circle is 8 cm.
Hence, the correct answer is option (c).
Page No 13.70:
Question 24: The area of the circle that can be inscribed in a square of
side 10 cm is

(a) 40 π cm2

(b) 30 π cm2

(c) 100 π cm2

T IO S I
(d) 25 π cm2
NO A D
CO N
A
ANSWER:

PY
DO U A
T
ED PR
C
©

We know that ABCD is a square of length 10 cm. A circle is inscribed in


the square therefore, all the sides of the square are become tangents of the
circle.
By, the tangent property, we have
𝐴𝑃 = 𝑃𝐷 = 5
𝐴𝑄 = 𝑄𝐵 = 5
𝐵𝑅 = 𝑅𝐶 = 5
𝐶𝑆 = 𝐷𝑆 = 5
If we join PR, then it will be the diameter of the circle of 10 cm.
Therefore, radius of the circle = 5cm
∴ Area of the circle = 𝜋𝑟 2
∴ Area of the circle = 𝜋 × 52
∴ Area of the circle = 25𝜋
Therefore, area of the circle is 25𝜋 cm².
Hence, the correct answer is option (d).

T IO S I
Page No 13.70:
NO A D
CO N
Question 25: If the difference between the circumference and radius of
A
a circle is 37 cm, then its area is

PY
DO U A

(a) 154 cm2


T
ED PR

(b) 160 cm2


C

(c) 200 cm2


©

(d) 150 cm2


ANSWER:
We have given the difference between circumference and radius of the
circle.
Let C be the circumference, r be the radius and A be the area of the circle.
Therefore, from the given condition we have
𝐶 − 𝑟 = 2𝜋𝑟 − 𝑟
∴ 37 = 2𝜋𝑟 − 𝑟
∴ 37 = 𝑟(2𝜋 − 1)
37
∴ 𝑟 = (2𝜋−1)
22
Now we will substitute 𝜋 = .
7
37
∴𝑟= 22
(2× 7 −1)

37
∴𝑟= 44
( 7 −1)

37
∴𝑟=

T IO S
44−7
( 7 )

I
∴𝑟= NO A D
37

CO N
37
(7)
A
7
∴ 𝑟 = 37 ×

PY
DO U A
37
T
∴𝑟=7
ED PR

Now we will substitute the value of r in 𝐴 = 𝜋𝑟 2 .


∴ 𝐴 = 𝜋 × 72
C
©

22
Now we will substitute 𝜋 = .
7
22
∴𝐴= × 72
7

∴ 𝐴 = 22 × 7
∴ 𝐴 = 154
Therefore, area of the circle is 154 cm².
Hence, the correct answer is option (a).

Page No 13.71:
Question 26:
The area of a circular path of uniform width h surrounding a circular
region of radius r is

(a) 𝜋(2𝑟 + ℎ)𝑟

(b) 𝜋(2𝑟 + ℎ)ℎ

(c) 𝜋(ℎ + 𝑟)𝑟

T IO S
(d) 𝜋(ℎ + 𝑟)ℎ

I
NO A D
ANSWER:

CO N
A

PY
DO U A
T
ED PR
C
©

We have
OA = r
AB = h
Therefore, radius of the outer circle will be 𝑟 + ℎ.
Now we will find the area between the two circles.
Area of the circular path = area of the outer circle – area of the inner
circle
∴ Area of the circular path = 𝜋(𝑟 + ℎ)2 − 𝜋𝑟 2
∴ Area of the circular path = 𝜋(𝑟 2 + 2𝑟ℎ + ℎ2 ) − 𝜋𝑟 2
∴ Area of the circular path = 𝜋(𝑟 2 + 2𝑟ℎ + ℎ2 − 𝑟 2 )
Cancelling 𝑟 2 we get,
Area of the circular path = 𝜋(2𝑟ℎ + ℎ2 )
∴ Area of the circular path = 𝜋(2𝑟 + ℎ)ℎ
Therefore, area of the circle is 𝜋(2𝑟 + ℎ)ℎ.

T IO S
Hence, the correct answer is option (b).

I
NO A D
CO N
Page No 13.71:
A
Question 27: If AB is a chord of length 5√3 cm of a circle with

PY
DO U A
centre O and radius 5 cm, then area of sector OAB is
T
ED PR

3𝜋
(a) cm2
8
C

8𝜋
(b) cm2
©

(c) 25𝜋 cm2

25𝜋
(d) cm2
3

ANSWER:
We have to find the area of the sector OAB.
T IO S
We have,

I
AM = NO A D
5√3

CO N
2
A
So,

PY
DO U A
5√3
sin ∠𝐴𝑂𝑀 =
T
2(5)
ED PR

Hence,
C

∠𝐴𝑂𝑀 = 60°
©

Therefore, area of the sector,


1
= 𝑟2𝜃
2
1 2𝜋
= (25) ( )
2 3
25𝜋
= cm²
3

So answer is (d)

Page No 13.71:
Question 28: The area of a circle whose area and circumference are
numerically equal, is
(a) 2π sq. units

(b) 4π sq. units

(c) 6π sq. units

(d) 8π sq. units


ANSWER:

T IO S I
We have given that circumference and area of a circle are numerically
equal. NO A D
CO N
A
Let it be x.

PY
DO U A
Let r be the radius of the circle, therefore, circumference of the circle
T
is 2𝜋𝑟 and area of the circle will be 𝜋𝑟 2 .
ED PR

Therefore, from the given condition we have,


……… (1)
C

2𝜋𝑟 = 𝑥
©

𝜋𝑟 2 = 𝑥 ……… (2)
𝑥
Therefore, from equation (1) get 𝑟 = . Now we will substitute this
2𝜋
𝑥 2
value in equation (2) we get, 𝑟 = ( ) = 𝑥
2𝜋

Simplifying further we get,


𝑥2
𝜋× =𝑥
4𝜋2

Cancelling x, we get,
𝑥
𝜋× =1
4𝜋2

Now we will cancel 𝜋


𝑥
=1 ……… (3)
4𝜋

Now we will multiply both sides of the equation (3) by 4𝜋 we get,


𝑥 = 4𝜋
Therefore, area of the circle is 4𝜋 sq. units.
Hence, option (b) is correct.

T IO S
Page No 13.71:

I
Question 29: If diameter of a circle is increased by 40%, then its area
NO A D
CO N
increase by
A

PY
(a) 96%
DO U A
T
ED PR

(b) 40%

(c) 80%
C
©

(d) 48%
ANSWER:
𝑑
If d is the original diameter of the circle, then the original radius is .
2

𝑑 2
∴ area of the circle = 𝜋 ( )
2
𝑑2
∴ area of the circle = 𝜋 ×
4

If diameter of the circle increases by 40%, then new diameter of the


circle is calculated as shown below,
That is new diameter = 𝑑 + 0.4𝑑
= 1.4𝑑
1.4𝑑
∴ new radium =
2

∴ new radium = 0.7𝑑


So, new area will be 𝜋(0.7𝑑)2 .
∴ new area = 𝜋 × 0.49𝑑2
Now we will calculate the change in area.

T IO S
𝑑2
∴ change in area = 𝜋 × 0.49𝑑2 − 𝜋 ×

I
4

NO A D
∴ change in area = (0.49 − ) 𝜋𝑑2
1

CO N
A
4
𝑑2

PY
∴ change in area = 0.96𝜋
DO U A
4
T
Therefore, its area is increased by 96%.
ED PR

Hence, the correct answer is option (a).


C
©

Page No 13.71:
Question 30: In the following figure, the shaded area is

(a) 50 (π−2) cm2

(b) 25 (π−2) cm2

(c) 25 (π+2) cm2

(d) 5 (π−2) cm2


T IO S
ANSWER:

I
Area of the shaded region is-
NO A D
CO N
𝜋𝜃 𝜃 𝜃
− sin cos ] (𝑟)2
A
=[
360 2 2

PY
𝜋 1
DO U A
= ( − ) (10)2
4 2
T
ED PR

= 25(𝜋 − 2) cm²
So the answer is (b).
C
©

Page No 13.71:
Question 31: In the following figure, the area of the segment PAQ is

𝑎2
(a) (𝜋 + 2)
4

𝑎2
(b) (𝜋 − 2)
4

𝑎2
(c) (𝜋 − 1)
4

𝑎2
(d) (𝜋 + 1)
4
T IO S I
ANSWER:
NO A D
CO N
We have to find area of segment PAQ.
A
𝜋𝜃 𝜃 𝜃

PY
Area of the PAQ segment = ( − sin cos ) 𝑟 2
DO U A
360 2 2
T
We know that 𝜃 = 90°.
ED PR

Substituting the values, we get,


C

𝜋×90
Area of the PAQ segment = ( − sin 45 cos 45) 𝑎2
©

360
𝜋
∴ Area of the PAQ segment = ( − sin 45 cos 45) 𝑎2
4
1 1
Substituting sin 45 = and cos 45 = we get,
√2 √2
𝜋 1 1
Area of the PAQ segment = ( − × ) 𝑎2
4 √2 √2
𝜋 1
∴ Area of the PAQ segment = ( − ) 𝑎2
4 2

Now we will make the denominator same.


𝜋 2
∴ Area of the PAQ segment = ( − ) 𝑎2
4 4
𝜋−2
∴ Area of the PAQ segment = ( ) 𝑎2
4
𝑎2
∴ Area of the PAQ segment = (𝜋 − 2)
4
𝑎2
∴ Area of the PAQ segment = (𝜋 − 2)
4
𝑎2
Therefore, area of the segment PAQ is (𝜋 − 2).
4

Hence, the correct answer is option (b).

T IO S
Page No 13.71:

I
Question 32: In the following figure, the area of segment ACB is
NO A D
CO N
A
𝜋 √3
(a) ( − ) 𝑟2

PY
3 2
DO U A
T
𝜋 √3
) 𝑟2
ED PR

(b) ( +
3 2

𝜋 2
C

(c) ( − ) 𝑟2
3 √3
©

(d) None of these

ANSWER:
We have to find area of segment ACB.
𝜋𝜃 𝜃 𝜃
Area of the ABC segment = ( − sin cos ) 𝑟 2
360 2 2

We know that 𝜃 = 120°.


Substituting the values, we get,
𝜋×120
Area of the ABC segment = ( − sin 60 cos 60) 𝑟 2
360
𝜋
Area of the ABC segment = ( − sin 60 cos 60) 𝑟 2
3

√3 1
Substituting sin 60 = and cos 60 = we get,
2 2
𝜋 √3 1
Area of the ABC segment = ( − × ) 𝑟2
3 2 2
𝜋 √3
∴ Area of the ABC segment = ( − ) 𝑟2
3 4

T IO S
𝜋 √3
) 𝑟2.

I
Therefore, area of the segment ACB is ( −
3 4
NO A D
CO N
Hence, the correct answer is option (d).
A

PY
DO U A

Page No 13.72:
T
ED PR

Question 33: If the area of a sector of a circle bounded by an arc of length


5π cm is equal to 20π cm2, then the radius of the circle
C

(a) 12 cm
©

(b) 16 cm

(c) 8 cm

(d) 10 cm
ANSWER:
We have given length of the arc and area of the sector bounded by that arc
and we are asked to find the radius of the circle.
If l is the length of the arc, A is the area of the arc and r is the radius of
the circle, then we know the expression of the area of the sector in terms
of the length of the arc and radius of the circle.
1
Area of the sector = 𝑙𝑟
2

Now we will substitute the corresponding values of length of the arc and
area of the sector.
1
∴ 20𝜋 = × 5𝜋 × 𝑟
2

Multiplying both sides of the equation by 2 we get,


40𝜋 = 5𝜋 × 𝑟

T IO S
Dividing both sides of the equation by 5𝜋 we get,

I
𝑟=8
NO A D
CO N
A
Therefore, radius of the circle is 8 cm.

PY
Hence, the correct answer is option (c).
DO U A
T
ED PR

Page No 13.72:
Question 34: In the following figure, the ratio of the areas of two sectors
C
©

S1 and S2 is
(a) 5 : 2

(b) 3 : 5

(c) 5 : 3

(d) 4 : 5

ANSWER:
𝜃1
Area of the sector, 𝑆1 = × 𝜋𝑟 2
360
𝜃2
Area of the sector, 𝑆2 = × 𝜋𝑟 2
360

Now we will take the ratio,


𝜃1
𝑆1 360
×𝜋𝑟 2
= 𝜃2
𝑆2 ×𝜋𝑟 2
360

Now we will simplify the ratio as below,


𝑆1 𝜃1
=
𝑆2 𝜃2

T IO S
Substituting the values, we get,

I
𝑆1
= NO A D
120

CO N
𝑆2 150
A
𝑆1 4

PY
∴ =
DO U A
𝑆2 5
T
Therefore, ratio of the areas of the two sectors is 4: 5.
ED PR

Hence, the correct answer is option (d).


C
©

Page No 13.72:
Question 35: If the area of a sector of a circle is 518 of the area of the
circle, then the sector angle is equal to
(a) 60°

(b) 90°

(c) 100°

(d) 120°
ANSWER:
5
We have given that area of the sector is of the area of the circle.
18
5
Therefore, area of the sector = × area of the circle
18
𝜃 5
∴ × 𝜋𝑟 2 = × 𝜋𝑟 2
360 18

Now we will simplify the equation as below,


𝜃 5
=
360 18

T IO S
Now we will multiply both sides of the equation by 360,

I
5
∴𝜃=
NO A D
18
× 360

CO N
A
∴ 𝜃 = 100

PY
DO U A
Therefore, sector angle is 100°.
T
Hence, the correct answer is option (c).
ED PR
C

Page No 13.72:
©

Question 36: If he area of a sector of a circle is 720 of the area of the


circle, then the sector angle is equal to

(a) 110°

(b) 130°

(c) 100°

(d) 126°
ANSWER:
7
We have given that area of the sector is of the area of the circle.
20
7
Therefore, area of the sector = × area of the circle
20
𝜃 7
∴ × 𝜋𝑟 2 = × 𝜋𝑟 2
360 20

Now we will simplify the equation as below,


𝜃 7
=
360 20

Now we will multiply both sides of the equation by 360,


7

T IO S
∴𝜃= × 360
20

I
∴ 𝜃 = 126
NO A D
CO N
A
Therefore, sector angle is 126°.

PY
Hence, the correct answer is option (d).
DO U A
T
ED PR

Page No 13.72:
Question 37: In the following figure, If ABC is an equilateral triangle,
C
©

then shaded area is equal to

𝜋 √3
(a) ( − ) 𝑟2
3 4

𝜋 √3
(b) ( − ) 𝑟2
3 2

𝜋 √3
(c) ( + ) 𝑟2
3 4

𝜋
(d) ( + √3) 𝑟 2
3

ANSWER:
We have given that ABC is an equilateral triangle.
∴ ∠𝐴 = 60°
1
As we know that, ∠𝐵𝐶𝐴 = 𝑚(∠𝐵𝑂𝐶 ).
2
1
∴ 60° = 𝑚(∠𝐵𝑂𝐶 )
2

𝑚(∠𝐵𝑂𝐶 ) = 120°
Area of the shaded region = area of the segment BC.

T IO S
Let ∠𝐵𝑂𝐶 = 𝜃

I
NO A D
∴ Area of the segment = (
𝜋𝜃 𝜃 𝜃
− sin cos ) 𝑟 2

CO N
360 2 2
A
Substituting the values, we get,

PY
DO U A
𝜋×120
Area of the segment = ( − sin 60 cos 60) 𝑟 2
T
360
ED PR

𝜋
∴ Area of the segment = ( − sin 60 cos 60) 𝑟 2
3
C

√3 1
Substituting sin 60 = and cos 60 = we get,
©

2 2
𝜋 1 √3
Area of the segment = ( − × ) 𝑟2
3 2 2
𝜋 √3
∴ Area of the segment = ( − ) 𝑟2
3 4
𝜋 √3
Therefore, area of the shaded region is ( − ) 𝑟2. Hence, the correct
3 4
answer is option (a).

Page No 13.73:
Question 38: In the following figure, the area of the shaded region is

(a) 3π cm2
(b) 6π cm2

(c) 9π cm2

(d) 7π cm2

ANSWER:

T IO S
In the figure,

I
∠C = ∠B = 90° and ∠D = 60°
NO A D
CO N
∴ ∠A + ∠B + ∠C + ∠D = 360°
A

PY
∠A + 90° + 90° + 60° = 360°
DO U A

∴ ∠A = 120ׄ°
T
ED PR

𝜃
Area of shaded region = × 𝜋𝑟 2
360
C

120
∴ Area of shaded region = × 𝜋 × 32
©

360
1
∴ Area of shaded region = × 𝜋 × 9
3

∴ Area of shaded region = 3𝜋


Therefore, area of the shaded region is 3𝜋 cm².
Hence, the correct answer is option (a).

Page No 13.73:
Question 39: If the perimeter of a circle is equal to that of a square, then
the ratio of their areas is

(a) 13 : 22
(b) 14 : 11

(c) 22 : 13

(d) 11 : 14
ANSWER:
We have given that perimeter of circle of radius r is equal to square of

T IO S
side a.

I
∴ perimeter of the circle = perimeter of the square
NO A D
CO N
∴ 2𝜋𝑟 = 4𝑎
A
4𝑎

PY
∴𝑟=
DO U A
2𝜋
T
2𝑎
ED PR

∴𝑟=
𝜋

Now we will substitute value of r in the following equation


C

Area of circle 𝜋𝑟 2
©

=
Area of square 𝑎2

2𝑎 2
Area of circle 𝜋( )
𝜋
∴ =
Area of square 𝑎2

4𝑎2
Area of circle 𝜋× 2
𝜋
∴ =
Area of square 𝑎2

4𝑎2
Area of circle 𝜋
∴ =
Area of square 𝑎2

Area of circle 4𝑎2 1


∴ = ×
Area of square 𝜋 𝑎2
Area of circle 4𝑎
∴ =
Area of square 𝜋
22
Substituting 𝜋 = we get,
7
Area of circle 4𝑎
∴ = 22
Area of square
7

Area of circle 28
∴ =
Area of square 22
Area of circle 14
∴ =
Area of square 11

Hence, ratio of the areas of the circle and square is 14: 11.

T IO S
Therefore, the correct answer is (b).

I
NO A D
CO N
A
Page No 13.73:

PY
DO U A
Question 40: The radius of a circle is 20 cm. It is divided into four parts
T
of equal area by drawing three concentric circles inside it. Then, the radius
ED PR

of the largest of three concentric circles drawn is


(a) 10√5 cm
C
©

(b) 10√3 cm

(c) 10 cm

(d) 10√2 cm
ANSWER:
The circle can be divided into four parts of equal area by drawing three
concentric circles inside it as,
T IO S
It is given that OB = 20 cm. Let OA = x.

I
NO A D
Since the circle is divided into four parts of equal area by the three

CO N
concentric circles, we have,
A
1

PY
Area of the fourth region = × Area of the given circle
DO U A
4
T
1
𝜋 × (202 − 𝑥 2 ) = × 𝜋 × 202
ED PR

400 − 𝑥 2 = 100
C

𝑥 2 = 300
©

𝑥 = 10√3 cm
Therefore, the correct answer is (b).

Page No 13.73:
Question 41: The area of a sector whose perimeter is four times its
radius r units, is

𝑟2
(a) sq. units
4

(b) 2r2 sq. units


(c) r2 sq. units

𝑟2
(d) sq. units
2

ANSWER:
𝜃
We know that perimeter of the sector = 2𝑟 + × 2𝜋𝑟.
360

We have given that perimeter of the sector is four times the radius.

T IO S
𝜃

I
2𝑟 + × 2𝜋𝑟 = 4𝑟
NO A D
360

CO N
Subtracting 2r from both sides of the equation,
A
𝜃

PY
∴ × 2𝜋𝑟 = 4𝑟 − 2𝑟
DO U A
360
T
𝜃
ED PR

∴ × 2𝜋𝑟 = 2𝑟
360

Dividing both sides of the equation 2r we get,


C

𝜃
©

×𝜋 =1
360
𝜃𝜋
∴ = 1 ………. (1)
360

Let us find the area of the sector.


𝜃
∴ Area of the sector = 𝜋𝑟 2
360
𝜃𝜋
Substituting = 1 we get,
360

Area of the sector = 𝑟 2


Hence, area of the sector is r² sq. units.
Therefore, the correct answer is (c).
Page No 13.73:
Question 42: If a chord of a circle of radius 28 cm makes an angle of 90°
at the centre, then the area of the major segment is
(a) 392 cm2

(b) 1456 cm2

(c) 1848 cm2

T IO S I
(d) 2240 cm2
NO A D
CO N
ANSWER:
A
Area of major segment,

PY
DO U A
𝜋𝜃 𝜃 𝜃
= Area of circle = [ − sin cos ] (𝑟)2
T
360 2 2
ED PR

𝜋 1
= 𝜋(28)2 − ( − ) (28)2
4 2
C

= 784𝜋 − 196(𝜋 − 2)
©

= 2240 cm²
So the answer is (d)

Page No 13.73:
Question 43: If area of a circle inscribed in an equilateral triangle is 48π
square units, then perimeter of the triangle is

(a) 17√3 units

(b) 36 units
(c) 72 units

(d) 48√3 units


ANSWER:
Let the circle of radius r be inscribed in an equilateral triangle of side a.

T IO S I
NO A D
CO N
A

PY
DO U A
T
ED PR

Area of the circle is given as 48π.


⇒ πr2 = 48π
C
©

⇒ r2 = 48
⇒ 𝑟 = 4√3
Now, it is clear that ON⊥BC. So, ON is the height of ΔOBC
corresponding to BC.
Area of ΔABC = Area of ΔOBC + Area of ΔOCA + Area of ΔOAB = 3
× Area of ΔOBC
√3 1
× 𝑎2 = 3 × × 𝐵𝐶 × 𝑂𝑁
4 2
√3 1
× 𝑎2 = 3 × × 𝑎 × 𝑟
4 2
√3 1
× 𝑎2 = 3 × × 𝑎 × 4√3
4 2
𝑎 = 24
Thus, perimeter of the equilateral triangle = 3 × 24 units = 72 units
So the answer is (c).

Page No 13.74:
Question 44: The hour hand of a clock is 6 cm long. The area swept by
it between 11.20 am and 11.55 am is

T IO S I
(a) 2.75 cm2
NO A D
CO N
A
(b) 5.5 cm2

PY
DO U A
(c) 11 cm2
T
ED PR

(d) 10 cm2
C

ANSWER:
©

1
Hour hand moves ( °)in one minute.
2

So, area,
1 𝜃
= (𝑟 2 ) ( 𝜋)
2 180
1 35
= (36) ( 𝜋)
2 2(180)

= 5.5 cm²
So the answer is (b)

Page No 13.74:
Question 45: ABCD is a square of side 4 cm. If E is a point in the interior
of the square such that ΔCED is equilateral, then area of Δ ACE is
(a) 2√3 − 1 cm2

(b) 4√3 − 1 cm2

(c) 63√3 − 1 cm2

T IO S
(d) 8√3 − 1 cm2

I
NO A D
ANSWER:

CO N
A
We have the following diagram.

PY
DO U A
T
ED PR
C
©

Since ∆CED is equilateral,


Therefore,
EC = CD = DE = 4 cm
Now, ∠𝐸𝐶𝐷 = 60°
Since AC is diagonal of sq r. ABCD
Therefore,
∠𝐴𝐶𝐷 = 45°
Therefore, we get,
∠𝐸𝐶𝐴 = ∠𝐸𝐶𝐷 − ∠𝐴𝐶𝐷
= 60° − 45°
= 15°
Now, in Δ ACE, draw a perpendicular EM to the base AC.
So in Δ EMC,
𝐸𝑀

T IO S
sin 15° =
𝐸𝐶

I
𝐸𝑀
NO A D
=

CO N
4
A
Therefore,

PY
DO U A
EM = √2(√3 − 1)
T
ED PR

Now in ΔAEC,
1
𝑎𝑟(ΔA𝐸𝐶 ) = (𝐴𝐶 )(𝐸𝑀)
2
C
©

= 4(√3 − 1) cm²
So the answer is (b)

Page No 13.74:
Question 46: If the area of a circle is equal to the sum of the areas of two
circles of diameters 10 cm and 24 cm, then diameter of the large circle (in
cm) is
(a) 34

(b) 26

(c) 17
(d) 14
ANSWER:
Let the diameter of the larger circle be 𝑑
Now, Area of larger circle = Area of circle having diameter 10 cm +
Area of circle having diameter 24 cm
𝑑 2 10 2 24 2
⇒ 𝜋( ) = 𝜋( ) +𝜋( )
2 2 2

T IO S
𝑑 2
⇒ ( ) = (5)2 + (12)2

I
2

NO A D
𝑑 2

CO N
⇒ ( ) = 25 + 144
A
2

PY
𝑑 2
DO U A
⇒ ( ) = 132
2
T
ED PR

𝑑
⇒ = 13
2

⇒ 𝑑 = 26 cm
C
©

Hence, the correct answer is option (b).

Page No 13.74:
22
Question 47: If π is taken as , the distance (in metres) covered by a
7
wheel of diameter 35 cm, in one revolution, is

(a) 2.2

(b) 1.1

(c) 9.625
(d) 96.25
ANSWER:
The distance covered by the wheel in one revolution is equal to the
circumference of the wheel.
Circumference = 𝜋𝑟 2
22 35 2
= ( )
7 2

T IO S
22 35 2

I
= ( )
7 2
NO A D
CO N
= 962.5 cm
A
The distance covered by the wheel in one revolution (in m) is given by

PY
DO U A
962.5
= 9.625 m
100
T
ED PR

Hence, the correct answer is option (c).


C

Page No 13.74:
©

Question 48: ABCD is a rectangle whose three vertices are B (4,0), C


(4,3) and D (0, 3). The length of one of its diagonals is [CBSE 2014]
(a) 5
(b) 4
(c) 3
(d) 25
ANSWER:

𝐵𝐷 = √ (0 − 4)2 + (3 − 0)2
= √42 + 32
= √25
= 5 units
Hence, the correct answer is option (a).

Page No 13.74:
Question 49: Area of the largest triangle that can be inscribed in a semi-
circle of radius r units is
(a) r2 sq. units

T IO S
(b) 1212 r2 sq. units

I
NO A D
(c) 2 r2 sq. units

CO N
A
(d) 2–√2 r2 sq. units

PY
ANSWER:
DO U A
T 1
Area of a triangle = × base × height
ED PR

2
1
⇒ Area = × (2r) × r = r 2
2
C

So, the area of the largest triangle that can be inscribed in a semi-circle
©

of radius r is r2
Hence, the correct answer is option (a).

Page No 13.74:
Question 50: If the sum of the areas of two circles with radii 𝑟1 and 𝑟2 is
equal to the area of circle of radius rr, then
(a) 𝑟 = 𝑟1 + 𝑟2
(b) 𝑟12 + 𝑟22 = 𝑟 2
(c) 𝑟1 + 𝑟2 < 𝑟
(d) 𝑟12 + 𝑟22 < 𝑟 2
ANSWER:
The radius of the two circles are 𝑟1 and 𝑟2 .
Now, according to the given condition
Area of circle with radius r = Area of circle with radius 𝑟1 + Area of
circle with radius 𝑟2
𝜋𝑟 2 = 𝑟12 + 𝑟22
⇒ 𝑟2 = 𝑟12 + 𝑟22
Hence, the correct answer is option (b).

T IO S
Page No 13.74:

I
NO A D
Question 51: If the sum of the circumferences of the two circles with

CO N
A
radii 𝑟1 and 𝑟2 is equal to the circumference of a circle of radius rr, then

PY
(a) 𝑟 = 𝑟1 + 𝑟2
DO U A
T
(b) 𝑟1 + 𝑟2 > 𝑟
ED PR

(c) 𝑟1 + 𝑟2 < 𝑟
C

(d) None of these


©

ANSWER:
The circumference of the circle with radius r = circumference of the circle
with radius 𝑟1 + circumference of the circle with radius 𝑟2
2𝜋𝑟 = 2𝜋𝑟1 + 2𝜋𝑟2
⇒ 𝑟 = 𝑟1 + 𝑟2
Hence, the correct answer is option (a).

Page No 13.74:
Question 52: If the circumference of a circle and the perimeter of a square
are equal, then
(a) Area of the circle = Area of the square
(b) Area of the circle < Area of the square
(c) Area of the circle > Area of the square
(d) nothing definite can be said
ANSWER:
Circumference of the circle = Perimeter of the square
2𝜋𝑟 = 4𝑎

T IO S
2𝑎
⇒𝑟=

I
𝜋
NO A D 2𝑎 2 4𝑎2

CO N
Now area of the circle = 𝜋𝑟 2 = 𝜋 ( ) =
A
𝜋 𝜋
2
Area of the square = a

PY
DO U A
So, the area of the circle > area of the square
T
Hence, the correct answer is option (c).
ED PR

Page No 13.74:
C
©

Question 53: If the perimeter of a circle is equal to that of a square, then


the ratio of their areas is
(a) 22: 7
(b) 14: 11
(c) 7: 22
(d) 11: 14
ANSWER:
Perimeter of a circle = Perimeter of a square
⇒ 2𝜋𝑟 = 4𝑎
2𝑎
⇒𝑟=
𝜋
2 2𝑎 2 4𝑎2
Now area of the circle = 𝜋𝑟 = 𝜋 ( ) =
𝜋 𝜋

Area of the square = a2

4𝑎2
Area of circle 𝜋 4 14
= = 22 =
Area of square 𝑎2 11
7

T IO S
Hence, the required ratio is 14: 11.

I
NO A D
The correct answer is option (b).

CO N
A

PY
DO U A
T
ED PR
C
©

You might also like